Корень в корне формула: Что такое квадратный корень? Формулы и Примеры

Содержание

Что такое квадратный корень? Формулы и Примеры

Статья находится на проверке у методистов Skysmart.
Если вы заметили ошибку, сообщите об этом в онлайн-чат
(в правом нижнем углу экрана).

Что такое квадратный корень

Определение арифметического квадратного корня ясности не добавляет, но заучить его стоит:

Арифметическим квадратным корнем из неотрицательного числа a называется такое неотрицательное число, квадрат которого равен a.

Определение квадратного корня также можно представить в виде формул:
√a = x
x2 = a
x ≥ 0
a ≥ 0

Из определения следует, что a не может быть отрицательным числом. То есть то, что стоит под корнем — обязательно положительное число.

Чтобы разобраться, почему именно так и никак иначе, давайте рассмотрим пример.

Попробуем найти корень из √-16

Здесь логично предположить, что 4, но давайте проверим: 4*4 = 16 — не сходится.

Если — 4, то -4 * -4 = 16, (минус на минус всегда дает плюс).

Получается, что ни одно число не может дать отрицательный результат при возведении его в квадрат.

Числа, стоящие под знаком корня, должны быть положительными.

Исходя из определения, значение корня также не должно быть отрицательным

Здесь могут возникнуть резонные вопросы, почему, например, в примере x2 = 16, x = 4 и x = -4.

Разница между квадратным корнем и арифметическим квадратным уравнением

Прежде всего, чтобы разграничить эти два понятия, запомните:

  • x2 = 16 не равно  x = √16.

Это два нетождественных друг другу выражения.

  • x2 = 16 — это квадратное уравнение.
  • x = √ 16 — арифметический квадратный корень.

Из выражения x2 = 16 следует, что:

  • |x| = √16, это значит, что x = ±√16 = ±4, x1 = 4, x2 = -4.

Если две вертикальные палочки возле x вводят вас в замешательство, почитайте нашу статью о модуле числа.

В то же самое время, из выражения x = √16 следует, что x = 4.

Если ситуация все еще кажется запутанной и нелогичной, просто запомните, что отрицательное число может быть решением только в квадратном уравнении. Если в решении «минус» — есть два варианта:

 
  1. Пример решен неверно

  2. Это квадратное уравнение.

Если вы извлекаете квадратный корень из числа, то можете быть уверены, вас ждет «положительный» результат.

Давайте рассмотрим пример, чтобы окончательно выяснить разницу между квадратным корнем и квадратным уравнением.

Даны два выражения: 

 
  1. x2 = 36

  2. x = √36

Первое выражение — квадратное уравнение. 

|x| = √36
x1 = +6
x2 = -6.

Второе выражение — арифметический квадратный корень. 

√36 = 6
x = 6.

Мы видим, что результатом решения первого выражения стали два числа — отрицательное и положительное. А во втором случае — только положительное.

Запись иррациональных чисел с помощью квадратного корня

Иррациональное число — это число, которое нельзя представить в виде обыкновенной дроби.

Чаще всего, иррациональные числа можно встретить в виде корней, логарифмов, степеней и т.д.

Примеры иррациональных чисел:

√2 = 1,414213…;

π = 3,141592…;

e = 2,718281…. .

Чтобы упростить запись иррациональных чисел, математики ввели понятие квадратного корня. Давайте разберем пару примеров, чтобы увидеть квадратный корень в деле.

Дано уравнение: x2 = 2.

Сразу сталкиваемся с проблемой, поскольку очевидно, что ни одно целое число не подходит. 

Переберем числа, чтобы удостовериться в этом:

1 * 1 = 1,
2 * 2 = 4,
3 * 3 = 9.

Отрицательные числа дают такой же результат. Значит результатом решения не могут быть целые числа.

Решение следующее:
Строим график функции y = x2.
Отмечаем решения на графике: -√2; √2.


Если попробовать извлечь квадратный корень из 2 с помощью калькулятора, то результат будет следующий: √2 = 1,414213… .

В таком виде ответ не записывают — нужно оставить квадратный корень.

x2 = 2.
x = √2
x = -√2. 

Извлечение корней

Решать примеры с квадратными корнями намного легче, если запомнить как можно больше квадратов чисел. Для этого воспользуйтесь таблицей — сохраните ее себе и используйте для решения задачек.

Таблица квадратов


Вот несколько примеров извлечения корней, чтобы научиться пользоваться таблицей:

  • 1. Извлеките квадратный корень: √289

Ищем в таблице число 289, двигаемся от него влево и вверх, чтобы определить цифры, образующие нужное нам число.

Влево — 1, вверх — 7.

Ответ: √289 = 17.

  • 2. Извлеките квадратный корень: √3025

Ищем в таблице число 3025.
Влево — 5, вверх —  5.

Ответ: √3025 = 55.

  • 3. Извлеките квадратный корень: √7396

Ищем в таблице число 7396.

Влево — 8, вверх — 6.

Ответ: √7396 = 86.

  • 4. Извлеките корень: √9025

Ищем в таблице число 9025.

Влево — 9, вверх — 5.

Ответ: √9025 = 95.

  • 5. Извлеките корень √1600

Ищем в таблице число 1600.

Влево — 4, вверх — 0.

Ответ: √1600 = 40.

Извлечением корня называется нахождение его значение.

Свойства арифметического квадратного корня

У арифметического квадратного корня есть 3 свойства — их нужно запомнить, чтобы проще решать примеры.

  • Корень произведения равен произведению корней
  • Извлечь корень из дроби — это извлечь корень из числителя и из знаменателя
  • Чтобы возвести корень в степень, нужно возвести в степень значение под корнем

Давайте потренируемся и порешаем примеры на все три операции с корнями. Не забывайте обращаться к таблице квадратов. Попробуйте решить примеры самостоятельно, а для проверки обращайтесь к ответам.

Умножение арифметических корней

Для умножения арифметических корней используйте формулу:

Примеры:

 

Внимательно посмотрите на второе выражение и запомните, как записываются такие примеры.

Если нет возможности извлечь корни из чисел, то поступаем так:

 

  1. Если множителей больше двух, то решается примерно точно так, как и с двумя множителями:

Добрая напоминалочка

Чтобы решать примеры быстрее, не забывайте пользоваться таблицей квадратов.

 


Деление арифметических корней

Для деления арифметических корней используйте формулу:

Примеры:

 
  1. Ответ: смешанную дробь превращаем в неправильную (16 * 3) + 1 = 49





Выполняя деление, не забывайте сокращать множители. При делении арифметических корней, используйте правила преобразования обыкновенных дробей.

Возведение арифметических корней в степень

Для возведения арифметического корня в степень используйте формулу:

Примеры:



Эти две формулы нужно запомнить:


Повторите свойства степеней или запишитесь на курсы по математике, чтобы без труда решать такие примеры.

Внесение множителя под знак корня

Вы уже умеете по-всякому крутить и вертеть квадратными корнями: умножать, делить, возводить в степень. Богатый арсенал, не правда ли? Осталось овладеть еще парой приемов и можно без страха браться за любую задачку.

А теперь давайте разберемся, как вносить множитель под знак корня.

Дано выражение: 7√9

Число семь умножено на квадратный корень из числа девять. 

Извлечем квадратный корень и умножим его на 7.

√9= 3.

7√9 = 7*3 = 21

В данном выражение число 7 — множитель. Давайте внесем его под знак корня. 

Запомните, что вносить множитель под знак корня обязательно нужно так, чтобы значение исходного выражения осталось неизменным. Иными словами, после наших манипуляций с корнем, значение выражения должно по-прежнему оставаться 21.

Вы помните, что (√a)2 = a

Тогда число 7 должно быть возведено во вторую степень. В этом случае значение выражения останется тем же. 

7√9 = √72* 9 = √49 * 9 = √49 * √9 = 7 * 3 = 21.

Формула внесения множителя под знак корня:

Запоминаем:

Нельзя вносить отрицательные числа под знак корня.

Потренируемся вносить множители. Попробуйте решить примеры самостоятельно, сверяясь с ответами.

 


Вынесение множителя из-под знака корня 

С тем, как вносить множитель под корень мы, кажется, разобрались. Но алгебра — такая алгебра, поэтому теперь неплохо бы и вынести множитель из-под знака корня.

Дано выражение в виде квадратного корня из произведения.

Вы уже наверняка без труда извлекаете квадратный корень из чего угодно, поэтому знаете, что делать.

Извлекаем корень из всех имеющихся множителей. 


В данном выражении квадратный корень мы можем извлечь только из 4, поэтому:


Таким образом множитель выносится из-под знака корня.

Давайте разберем примеры. Попробуйте вынести множители из-под знака корня самостоятельно, сверяясь с ответами.

 
  1. √28

    Раскладываем подкоренное выражение на множители 28 = 7*4.

    Извлекаем корень из 4. Множитель 7 оставляем под знаком корня.



  2. Ответ: по правилу извлечения квадратного корня из произведения,

    Так как вынесенный множитель должен стоять перед подкоренным знаком, то меняем их местами.

  3. Вынесите множитель из-под знака корня в выражении: √24

    Ответ: Раскладываем выражение под корнем на множители 24 = 6 * 4.


  4. Упростите выражение:

    Вынесем в двух последних выражения множитель из-под знака корня.

    Умножаем (-4 * 4) = -16. Все остальное выражение записываем в неизменном виде.

    Мы видим, что во всем выражении есть один общий множитель — √5.
    Выносим общий множитель за скобки:

    Далее вычисляем все, что в скобках:

 

Сравнение квадратных корней

Мы почти досконально разобрали арифметический квадратный корень, научились умножать, делить и возводить его в степень. Теперь вы без труда можете вносить множители под знак корня и выносить их оттуда. Осталось научиться сравнивать корни и стать непобедимым теоретиком.

Итак, чтобы понять, как сравнить два квадратных корня, нужно запомнить пару правил.

Если:

  • √a < √b, то a < b
  • √a = √b, то a = b

Давайте разберем на примере.

Сравните два выражения: √70 и 8√2

Первым делом преобразуем второе выражение: 8√2 = √64 * √2 = √64*2 = √128.

70 < 128.

Это значит, что √70  <  8√2.

Запоминаем

Чем больше число под знаком корня, тем больше сам корень.

Потренируйтесь в сравнении корней. Сверяете свои результаты с ответами.

 
  1. Сравните два выражения: √50 и 9√5

    Ответ: преобразовываем выражение 9√5.

    9√5 = √81 * √5 = √81*5 = √405

    50 < 405

    Это значит, что √50 < 9√5.


  2. Сравните два выражения: 6√5 и √18

    Ответ: преобразовываем выражение 6√5.

    6√5 = √36 * √5 = √36*5= √180

    180 > 18

    Это значит, что 6√5 > √18.


  3. Сравните два выражения: 7√12 и √20

    Ответ: преобразовываем выражение 7√12.

    7√12 = √49 * √12 = √49*12 = √588

    588 >20

    Это значит, что 7√12 > √20.

Как видите, ничего сложного в сравнении арифметических квадратных корней нет. 

Самое главное — выучить формулы и сверяться с таблицей квадратов, если значения корня слишком большие для легкого вычисления в уме.

Не бойтесь пользоваться вспомогательными материалами. Математика просто создана для того, чтобы окружить себя подсказками и намеками.

Когда вы почувствуете, что уже достаточно натренировались в решении примеров с квадратными корнями, можете позволить себе время от времени прибегать к помощи онлайн-калькуляторов. Они помогут решать примеры быстрее и быть эффективнее. 

Таких калькуляторов в интернете много, вот один из них.

Извлечение квадратного корня из большого числа

Вы уже наверняка познакомились и подружились с таблицей квадратов. Она — ваша правая рука. С ее помощью вы реактивно решаете примеры и, возможно, даже подумываете запомнить ее наизусть.

Но, как вы можете заметить, таблица заканчивается на числе 9801. А это, согласитесь, не самое крупное число из тех, что могут вам попасться в примере.


Чтобы извлечь корень из большого числа, которое отсутствует в таблице квадратов, нужно:

 
  1. Определить «сотни», между которыми оно стоит.

  2. Определить «десятки», между которыми оно стоит.

  3. Определить последнюю цифру в этом числе.

Извлечь корень из большого числа можно разными способами — вот один из них.

Извлечем корень из √2116.

Наша задача в том, чтобы определить между какими десятками стоит число 2116.

102 = 100

202 = 400

302 = 900

402 = 1600

502 = 2500 

Мы видим что, 2116 больше 1600, но меньше 2500.

Это значит, что число 2116 находится между 402и 502.

41, 42, 43, 44, 45, 46, 47, 48, 49.

Запомните лайфхак по вычислению всего на свете, что нужно возвести в квадрат.

Не секрет, что на последнем месте в любом числе может стоять только одна цифра от 1 до 0.


Как пользоваться таблицей

12 = 1

22 = 4

32 = 9

42 = 16 ⇒ 6

52 = 25 ⇒ 5

62 = 36 ⇒ 6

72 = 49 ⇒ 9

82 = 64 ⇒ 4

92 = 81 ⇒ 1

Мы знаем, что число 41, возведенное в квадрат, даст число, на конце которого — цифра 1.

Число, 42, возведенное в квадрат, даст число, на конце которого — цифра 4.

Число 43, возведенное в квадрат, даст число, на конце которого — 9.

Такая закономерность позволяет нам без записи «перебрать» все возможные варианты, исключая те, которые не дают нужную нам цифру 6 на конце.

Таким образом, у нас остаются два варианта: 442 и 462.

Далее вычисляем: 44 * 44 = 1936.

46 * 46 = 2116.

Ответ: √2116 = 46

Если такой способ показался не до конца понятным — можно потратить чуть больше времени и разложить число на множители. Если решить все правильно, получим такой же результат. 

Еще пример. Извлечем корень из числа √11664

Разложим число 11664 на множители: 

11666 : 4 = 2916

2916 : 4 = 729

729 : 3 = 243

243 : 3 = 81

11664

4

2916

4

729

3

243

3

81

81

Запишем выражение в следующем виде:


Извлечь квадратный корень из большого числа гораздо проще с помощью калькулятора. Но знать парочку таких способов «на экстренный случай» точно не повредит. Например, для контрольной или ЕГЭ.

Чтобы закрепить все теоретические знания, давайте ещё немного поупражняемся в решении примеров на арифметические квадратные корни.
 

Квадратный корень

Предварительные навыки

Основные сведения

Чтобы найти площадь квадрата, нужно длину его стороны возвести во вторую степень.

Найдём площадь квадрата, длина стороны которого 3 см

S = 32 = 9 см2

Теперь решим обратную задачу. А именно, зная площадь квадрата определим длину его стороны. Для этого воспользуемся таким инструментом как кóрень. Корень бывает квадратный, кубический, а также n-й степени.

Сейчас наш интерес вызывает квадратный корень. По другому его называют кóрнем второй степени.

Для нахождения длины стороны нашего квадрата, нужно найти число, вторая степень которого равна 9. Таковым является число 3. Это число и является кóрнем.

Введём для работы с корнями новые обозначения.

Символ кóрня выглядит как . Это по причине того, что слово корень в математике употребляется как радикал. А слово радикал происходит от латинского radix (что в переводе означает корень). Первая буква слова radix это r впоследствии преобразилась в символ корня .

Под корнем располагáют подкореннóе выражение. В нашем случае подкоренным выражением будет число 9 (площадь квадрата)

Нас интересовал квадратный корень (он же корень второй степени), поэтому слева над корнем указываем число 2. Это число называют показателем корня (или степенью корня)

Получили выражение, которое читается так: «квадратный корень из числа 9». С этого момента возникает новая задача по поиску самогó корня.

Если число 3 возвести во вторую степень, то получится число 9. Поэтому число 3 и будет ответом:

Значит квадрат площадью 9 см2 имеет сторону, длина которой 3 см. Приведённое действие называют извлечéнием квадрáтного кóрня.

Нетрудно догадаться, что квадратным корнем из числа 9 также является отрицательное число −3. При его возведении во вторую степень тоже получается число 9

Получается, что выражение  имеет два значения: 3 и −3. Но длина стороны квадрата не может быть отрицательным числом, поэтому для нашей задачи ответ будет только один, а именно 3.

Вообще, квадратный корень имеет два противоположных значения: положительное и отрицательное.

Например, извлечём квадратный корень из числа 4

Это выражение имеет два значения: 2 и −2, поскольку при возведении этих чисел во вторую степень, получится один и тот же результат 4

Поэтому ответ к выражению вида  записывают с плюсом и минусом. Плюс с минусом означает, что квадратный корень имеет два противоположных значения.

Запишем ответ к выражению  с плюсом и минусом:


Определения

Дадим определение квадратному корню.

Квадратным корнем из числа a называют такое число b, вторая степень которого равна a.

То есть число b должно быть таким, чтобы выполнялось равенство ba. Число b (оно же корень) обозначается через радикал  так, что . На практике левая и правая часть поменяны местами и мы видим привычное выражение 

Например, квадратным корнем из числá 16 есть число 4, поскольку число 4 во второй степени равно 16

42 = 16

Корень 4 можно обозначить через радикал  так, что .

Также квадратным корнем из числá 16 есть число −4, поскольку число −4 во второй степени равно 16

(−4)2 = 16

Если при решении задачи интересует только положительное значение, то корень называют не просто квадратным, а арифметическим квадратным.

Арифметический квадратный корень из числá a — это неотрицательное число b (b ≥ 0), при котором выполняется равенство ba.

В нашем примере квадратными корнями из числá 16 являются корни 4 и −4, но арифметическим из них является только корень 4.

В разговорном языке можно использовать сокращение. К примеру, выражение  полностью читается так: «квадратный корень из числá шестнадцать», а в сокращённом варианте можно прочитать так: «корень из шестнадцати».

Не следует путать понятия корень и квадрат. Квадрат это число, которое получилось в результате возведения какого-нибудь числá во вторую степень. Например, числа 25, 36, 49 являются квадратами, потому что они получились в результате возведения во вторую степень чисел 5, 6 и 7 соответственно.

Корнями же являются числа 5, 6 и 7. Они являются теми числами, которые во второй степени равны 25, 36 и 49 соответственно.

Чаще всего в квадратных корнях показатель кóрня вообще не указывается. Так, вместо записи можно использовать запись. Если в учебнике по математике встретится корень без показателя, то нужно понимать, что это квадратный корень.

Квадратный корень из единицы равен единице. То есть справедливо следующее равенство:

Это по причине того, что единица во второй степени равна единице:

12 = 1

и квадрат, состоящий из одной квадратной единицы, имеет сторону, равную единице:

Квадратный корень из нуля равен нулю. То есть справедливо равенство , поскольку 0= 0.

Выражение вида  смысла не имеет. Например, не имеет смысла выражение , поскольку вторая степень любого числа есть число положительное. Невозможно найти число, вторая степень которого будет равна −4.

Если выражение вида  возвести во вторую степень, то есть если записать , то это выражение будет равно подкореннóму выражению a

Например, выражение  равно 4

Это потому что выражение  равно значению 2. Но это значение сразу возвóдится во вторую степень и получается результат 4.

Еще примеры:

Корень из квадрата числá равен модулю этого числá:

Например, корень из числá 5, возведённого во вторую степень, равен модулю числá 5

Если во вторую степень возвóдится отрицательное число, ответ опять же будет положительным. Например, корень из числá −5, возведённого во вторую степень, равен модулю числá −5. А модуль числа −5 равен 5

Действительно, если не пользуясь правилом , вычислять выражение  обычным методом — сначала возвести число −5 во вторую степень, затем извлечь полученный результат, то полýчим ответ 5

Не следует путать правило  с правилом . Правило  верно при любом a, тогда как правило  верно в том случае, если выражение  имеет смысл.

В некоторых учебниках знак корня может выглядеть без верхней линии. Выглядит это так:

Примеры: √4, √9, √16.

Мéньшему числу соответствует мéньший корень, а бóльшему числу соответствует бóльший корень.

Например, рассмотрим числа 49 и 64. Число 49 меньше, чем число 64.

49 < 64

Если извлечь квадратные корни из этих чисел, то числу 49 будет соответствовать меньший корень, а числу 64 — бóльший. Действительно, √49 = 7, а √64 = 8,

√49 < √64

Отсюда:

7 < 8


Примеры извлечения квадратных корней

Рассмотрим несколько простых примеров на извлечение квадратных корней.

Пример 1. Извлечь квадратный корень √36

Данный квадратный корень равен числу, квадрат которого равен 36. Таковым является число 6, поскольку 6= 36

√36 = 6


Пример 2. Извлечь квадратный корень √49

Данный квадратный корень равен числу, квадрат которого равен 49. Таковым является число 7, поскольку 7= 49

√49 = 7

В таких простых примерах достаточно знать таблицу умножения. Так, мы помним, что число 49 входит в таблицу умножения на семь. То есть:

7 × 7 = 49

Но 7 × 7 это 72

7= 49

Отсюда, √49 = 7.


Пример 3. Извлечь квадратный корень √100

Данный квадратный корень равен числу, квадрат которого равен 100. Таковым является число 10, поскольку 102 = 100

√100 = 10

Число 100 это последнее число, корень которого можно извлечь с помощью таблицы умножения. Для чисел, бóльших 100, квадратные корни можно находить с помощью таблицы квадратов.


Пример 3. Извлечь квадратный корень √256

Данный квадратный корень равен числу, квадрат которого равен 256. Чтобы найти это число, воспользуемся таблицей квадратов.

Нахóдим в таблице квадратов число 256 и двигаясь от него влево и вверх определяем цифры, которые образуют число, квадрат которого равен 256.

Видим, что это число 16. Значит √256 = 16.


Пример 4. Найти значение выражения 2√16

В данном примере число 2 умножается на выражение с корнем. Сначала вычислим корень √16, затем перемнóжим его с числом 2


Пример 7. Решить уравнение 

В данном примере нужно найти значение переменной x, при котором левая часть будет равна 4.

Значение переменной x равно 16, поскольку . Значит корень уравнения равен 16.

Примечание. Не следует путать корень уравнения и квадратный корень. Корень уравнения это значение переменной, при котором уравнение обращается в верное числовое равенство. А квадратный корень это число, вторая степень которого равна выражению, находящемуся под радикалом .

Подобные примеры решают, пользуясь определением квадратного корня. Давайте и мы поступим так же.

Из определения мы знаем, что квадратный корень  равен числу b, при котором выполняется равенство ba.

Применим равенство ba к нашему примеру . Роль переменной b у нас играет число 4, а роль переменной a — выражение, находящееся под корнем , а именно переменная x

В выражении 4x вычислим левую часть, полýчим 16 = x. Поменяем левую и правую часть местами, полýчим = 16. В результате приходим к тому, что нашлось значение переменной x.


Пример 8. Решить уравнение 

Перенесем −8 в правую часть, изменив знак:

Возведем правую часть во вторую степень и приравняем её к переменной x

Вычислим правую часть, полýчим 64 = x. Поменяем левую и правую часть местами, полýчим = 64. Значит корень уравнения  равен 64


Пример 9. Решить уравнение 

Воспользуемся определением квадратного корня:

Роль переменной b играет число 7, а роль переменной a — подкореннóе выражение 3 + 5x. Возведем число 7 во вторую степень и приравняем его к 3 + 5x

В выражении 72 = 3 + 5x вычислим левую часть полýчим 49 = 3 + 5x. Получилось обычное линейное уравнение. Решим его:

Корень уравнения  равен . Выполним проверку, подставив его в исходное уравнение:


Пример 10. Найти значение выражения 

В этом выражении число 2 умножается на квадратный корень из числа 49.

Сначала нужно извлечь квадратный корень и перемножить его с числом 2


Приближённое значение квадратного корня

Не каждый квадратный корень можно извлечь. Извлечь квадратный корень можно только в том случае, если удаётся найти число, вторая степень которого равна подкореннóму выражению.

Например, извлечь квадратный корень  можно, потому что удаётся найти число, вторая степень которого равна подкореннóму выражению. Таковым является число 8, поскольку 8= 64. То есть

А извлечь квадратный корень  нельзя, потому что невозможно найти число, вторая степень которого равна 3. В таком случае говорят, что квадратный корень из числа 3 не извлекается.

Зато можно извлечь квадратный корень из числа 3 приближённо. Извлечь квадратный корень приближённо означает найти значение, которое при возведении во вторую степень будет максимально близко к подкореннóму выражению.

Приближённое значение ищут с определенной точностью: с точностью до целых, с точностью до десятых, с точностью до сотых и так далее.

Найдём значение корня  приближённо с точностью до десятых. Словосочетание «с точностью до десятых» говорит о том, что приближённое значение корня  будет представлять собой десятичную дробь, у которой после запятой одна цифра.

Для начала найдём ближайшее меньшее число, корень которого можно извлечь. Таковым является число 1. Корень из этого числа равен самому этому числу:

√1 = 1

Аналогично находим ближайшее бóльшее число, корень которого можно извлечь. Таковым является число 4. Корень из этого числа равен 2

√4 = 2

√1 меньше, чем √4

√1 < √4

А √3 больше, чем √1 но меньше, чем √4. Запишем это в виде двойного неравенства:

√1 < √3 < √4

Точные значения корней √1 и √4 известны. Это числа 1 и 2

1 < √3 < 2

Тогда очевидно, что значение корня √3 будет представлять собой десятичную дробь, потому что между числами 1 и 2 нет целых чисел.

Для нахождения приближённого значения квадратного корня √3 будем проверять десятичные дроби, располагающиеся в интервале от 1 до 2, возводя их в квадрат. Делать это будем до тех пор пока не полýчим значение, максимально близкое к 3. Проверим к примеру дробь 1,1

1,12 = 1,21

Получился результат 1,21, который не очень близок к подкореннóму выражению 3. Значит 1,1 не годится в качестве приближённого значения квадратного корня √3, потому что оно малó.

Проверим тогда дробь 1,8

1,82 = 3,24

Получился результат 3,24, который близок к подкореннóму выражению, но превосходит его на 0,24. Значит 1,8 не годится в качестве приближенного значения корня √3, потому что оно великó.

Проверим тогда дробь 1,7

1,72 = 2,89

Получился результат 2,89, который уже близок к подкореннóму выражению. Значит 1,7 и будет приближённым значением квадратного корня √3. Напомним, что знак приближенного значения выглядит как ≈

√3 ≈ 1,7

Значение 1,6 проверять не нужно, потому что в результате получится число 2,56, которое дальше от трёх, чем значение 2,89. А значение 1,8, как было показано ранее, является уже большим.

В данном случае мы нашли приближенное значение корня √3 с точностью до десятых. Значение можно получить ещё более точно. Для этого его следует находить с точностью до сотых.

Чтобы найти значение с точностью до сотых проверим десятичные дроби в интервале от 1,7 до 1,8

1,7 < √3 < 1,8

Проверим дробь 1,74

1,742 = 3,0276

Получился результат 3,0276, который близок к подкореннóму выражению, но превосходит его на 0,0276. Значит значение 1,74 великó для корня √3.

Проверим тогда дробь 1,73

1,732 = 2,9929

Получился результат 2,9929, который близок к подкореннóму выражению √3. Значит 1,73 будет приближённым значением квадратного корня √3 с точностью до сотых.

Процесс нахождения приближённого значения квадратного корня продолжается бесконечно. Так, корень √3 можно находить с точностью до тысячных, десятитысячных и так далее:

√3 = 1,732 (вычислено с точностью до тысячных)

√3 = 1,7320 (вычислено с точностью до десятитысячных)

√3 = 1,73205 (вычислено с точностью до ста тысячных).

Ещё квадратный корень можно извлечь с точностью до целых. Приближённое значение квадратного корня √3 с точностью до целых равно единице:

√3 ≈ 1

Значение 2 будет слишком большим, поскольку при возведении этого числа во вторую степень получается число 4, которое больше подкоренного выражения. Нас же интересуют значения, которые при возведении во вторую степень равны подкореннóму выражению или максимально близки к нему, но не превосходят его.

В зависимости от решаемой задачи допускается находить значение, вторая степень которого больше подкоренного выражения. Это значение называют приближённым значением квадратного корня с избытком. Поговорим об этом подробнее.


Приближенное значение квадратного корня с недостатком или избытком

Иногда можно встретить задание, в котором требуется найти приближённое значение корня с недостатком или избытком.

В предыдущей теме мы нашли приближённое значение корня √3 с точностью до десятых с недостатком. Недостаток понимается в том смысле, что до значения 3 нам недоставало ещё некоторых частей. Так, найдя приближённое значение √3 с точностью до десятых, мы получили 1,7. Это значение является значением с недостатком, поскольку при возведении этого числа во вторую степень полýчим результат 2,89. Этому результату недостаёт ещё 0,11 чтобы получить число 3. То есть, 2,89 + 0,11 = 3.

С избытком же называют приближённые значения, которые при возведении во вторую степень дают результат, который превосходит подкореннóе выражение. Так, вычисляя корень √3 приближённо, мы проверили значение 1,8. Это значение является приближённым значением корня √3 с точностью до десятых с избытком, поскольку при возведении 1,8 во вторую степень, получаем число 3,24. Этот результат превосходит подкореннóе выражение на 0,24. То есть 3,24 − 3 = 0,24.

Приближённое значение квадратного корня √3 с точностью до целых тоже был найден с недостатком:

√3 ≈ 1

Это потому что при возведении единицы в квадрат получаем единицу. То есть до числа 3 недостаёт ещё 2.

Приближённое значение квадратного корня √3 с точностью до целых можно найти и с избытком. Тогда этот корень приближённо будет равен 2

√3 ≈ 2

Это потому что при возведении числа 2 в квадрат получаем 4. Число 4 превосходит подкореннóе выражение 3 на единицу. Извлекая приближённо квадратный корень с избытком желательно уточнять, что корень извлечен именно с избытком:

√3 ≈ 2 (с избытком)

Потому что приближённое значение чаще всего ищется с недостатком, чем с избытком.

Дополнительно следует упомянуть, что в некоторых учебниках словосочетания «с точностью до целых», «с точностью до десятых», с «точностью до сотых», заменяют на словосочетания «с точностью до 1», «с точностью до 0,1», «с точностью до 0,01» соответственно.

Так, если в задании сказано извлечь квадратный корень из числа 5 с точностью до 0,01, то это значит что корень следует извлекать приближённо с точностью до сотых:

√5 ≈ 2,23


Пример 2. Извлечь квадратный корень из числа 51 с точностью до 1

√51 ≈ 7


Пример 3. Извлечь квадратный корень из числа 51 с точностью до 0,1

√51 ≈ 7,1

Пример 4. Извлечь квадратный корень из числа 51 с точностью до 0,01

√51 ≈ 7,14


Границы, в пределах которых располагаются корни

Если исходное число принадлежит промежутку [1; 100], то квадратный корень из этого исходного числа будет принадлежать промежутку [1; 10].

Например, пусть исходным числом будет 64. Данное число принадлежит промежутку [1; 100]. Сразу делаем вывод, что квадратный корень из числа 64 будет принадлежать промежутку [1; 10]. Теперь вспоминаем таблицу умножения. Какое перемножение двух одинаковых сомножителей даёт в результате 64? Ясно, что перемножение 8 × 8, а это есть 8= 64. Значит квадратный корень из числа 64 есть 8


Пример 2. Извлечь квадратный корень из числа 49

Число 49 принадлежит промежутку [1; 100]. Значит квадратный корень будет принадлежать промежутку [1; 10]. Этим корнем будет число 7, поскольку 7= 49

√49 = 7


Пример 2. Извлечь квадратный корень из числа 1

Число 1 принадлежит промежутку [1; 100]. Значит квадратный корень будет принадлежать промежутку [1; 10]. Этим корнем будет число 1, поскольку 1= 1

√1 = 1


Пример 3. Извлечь квадратный корень из числа 100

Число 100 принадлежит промежутку [1; 100]. Значит квадратный корень будет принадлежать промежутку [1; 10]. Этим корнем будет число 10, поскольку 10= 100

√100 = 10

Понятно, что промежуток [1; 100] содержит ещё и числа, квадратные корни из которых не извлекаются. Для таких чисел корень нужно извлекать приближённо. Тем не менее, приближённый корень тоже будет располагаться в пределах промежутка [1; 10].

Например, извлечём квадратный корень из числа 37. Нет целого числа, вторая степень которого была бы равна 37. Поэтому извлекать квадратный корень следует приближённо. Извлечём его к примеру с точностью до сотых:

√37 ≈ 6,08

Для облегчения можно находить ближайшее меньшее число, корень из которого извлекается. Таковым в данном примере было число 36. Квадратный корень из него равен 6. И далее отталкиваясь от числа 6, можно находить приближённое значение корня √37, проверяя различные десятичные дроби, целая часть которых равна 6.

Квадраты чисел от 1 до 10 обязательно нужно знать наизусть. Ниже представлены эти квадраты:

12 = 1
22 = 4
32 = 9
42 = 16
52 = 25
62 = 36
72 = 49
82 = 64
92 = 81
102 = 100

И обратно, следует знать значения квадратных корней этих квадратов:

Если к любому числу от 1 до 10 в конце дописать ноль (или несколько нулей), и затем возвести это число во вторую степень, то в полученном числе будет в два раза больше нулей.

Например, 6= 36. Допишем к числу 6 один ноль, полýчим 60. Возведём число 60 во вторую степень, полýчим 3600

60= 3600

А если к числу 6 дописать два нуля, и возвести это число во вторую степень, то полýчим число, в котором четыре нуля. То есть в два раза больше нулей:

6002 = 360000

Тогда можно сделать следующий вывод:

Если исходное число содержит знакомый нам квадрат и чётное количество нулей, то можно извлечь квадратный корень из этого числа. Для этого следует извлечь корень из знакомого нам квадрата и затем записать половину количества нулей из исходного числа.

Например, извлечём квадратный корень из числа 900. Видим, что в данном числе есть знакомый нам квадрат 9. Извлекаем из него корень, получаем 3

Теперь из исходного числа записываем половину от количества нулей. В исходном числе 900 содержится два нуля. Половина этого количества нулей есть один ноль. Записываем его в ответе после цифры 3


Пример 2. Извлечём квадратный корень из числа 90000

Здесь опять же имеется знакомый нам квадрат 9 и чётное количество нулей. Извлекаем корень из числа 9 и записываем половину от количества нулей. В исходном числе содержится четыре нуля. Половиной же этого количества нулей будет два нуля:


Пример 3. Извлечем квадратный корень из числа 36000000

Здесь имеется знакомый нам квадрат 36 и чётное количество нулей. Извлекаем корень из числа 36 и записываем половину от количества нулей. В исходном числе шесть нулей. Половиной же будет три нуля:


Пример 4. Извлечем квадратный корень из числа 2500

Здесь имеется знакомый нам квадрат 25 и чётное количество нулей. Извлекаем корень из числа 25 и записываем половину от количества нулей. В исходном числе два нуля. Половиной же будет один ноль:


Если подкореннóе число увеличить (или уменьшить) в 100, 10000 то корень увеличится (или уменьшится) в 10, 100 раз соответственно.

Например, . Если увеличим подкореннóе число в 100 раз, то квадратный корень увеличится в 10 раз:

И наоборот, если в равенстве  уменьшим подкореннóе число в 100 раз, то квадратный корень уменьшится в 10 раз:

Пример 2. Увеличим в равенстве  подкореннóе число в 10000, тогда квадратный корень 70 увеличиться в 100 раз

Пример 3. Уменьшим в равенстве  подкореннóе число в 100 раз, тогда квадратный корень 70 уменьшится в 10 раз

Эта закономерность позволяет извлечь квадратный корень из десятичной дроби, если в данной дроби после запятой содéржатся две цифры, и эти две цифры образуют знакомый нам квадрат. В таких случаях данную десятичную дробь следует умножить на 100. Затем извлечь квадратный корень из получившегося числа и уменьшить подкореннóе число в сто раз.

Например, извлечём квадратный корень из числа 0,25. В данной десятичной дроби после запятой содержатся две цифры и эти две цифры образуют знакомый нам квадрат 25.

Умнóжим десятичную дробь 0,25 на 100, полýчим 25. А из числа 25 квадратный корень извлекается легко:

Но нам изначально нужно было извлечь корень из 0,25, а не из 25. Чтобы исправить ситуацию, вернём нашу десятичную дробь. Если в равенстве  подкореннóе число уменьшить в 100 раз, то полýчим под корнем 0,25 и соответственно ответ уменьшится в 10 раз:

Обычно в таких случаях достаточно уметь передвигáть запятую. Потому что сдвинуть в числе запятую вправо на две цифры это всё равно что умножить это число на 100.

В предыдущем примере в подкоренном числе 0,25 можно было сдвинуть запятую вправо на две цифры, а в полученном ответе сдвинуть её влево на одну цифру.

Например, извлечем корень из числа 0,81. Мысленно передвинем запятую вправо на две цифры, полýчим 81. Теперь извлечём квадратный корень из числа 81, полýчим ответ 9. В ответе 9 передвинем запятую влево на одну цифру, полýчим 0,9. Значит, .

Это правило работает и в ситуации, когда после запятой содержатся четыре цифры и эти цифры образуют знакомый нам квадрат.

Например, десятичная дробь 0,1225 содержит после запятой четыре цифры. Эти четыре цифры образуют число 1225, квадратный корень из которого равен 35.

Тогда можно извлечь квадратный корень и из 0,1225. Умнóжим данную десятичную дробь на 10000, полýчим 1225. Из числа 1225 квадратный корень можно извлечь с помощью таблицы квадратов:

Но нам изначально нужно было извлечь корень из 0,1225, а не из 1225. Чтобы исправить ситуацию, в равенстве  подкореннóе число уменьшим в 10000 раз. В результате под корнем образуется десятичная дробь 0,1225, а правая часть уменьшится в 100 раз

Эта же закономерность будет работать и при извлечении корней из дробей вида 12,25. Если цифры из которых состоит десятичная дробь образуют знакомый нам квадрат, при этом после запятой содержится чётное количество цифр, то можно извлечь корень из этой десятичной дроби.

Умнóжим десятичную дробь 12,25 на 100, полýчим 1225. Извлечём корень из числа 1225

Теперь в равенстве уменьшим подкореннóе число в 100 раз. В результате под корнем образуется число 12,25, и соответственно ответ уменьшится в 10 раз


Если исходное число принадлежит промежутку [100; 10000], то квадратный корень из этого исходного числа будет принадлежать промежутку [10; 100].

В этом случае применяется таблица квадратов:

Например, пусть исходным числом будет 576. Данное число принадлежит промежутку [100; 10000]. Сразу делаем вывод, что квадратный корень из числа 576 будет принадлежать промежутку [10; 100]. Теперь открываем таблицу квадратов и смотрим какое число во второй степени равно 576

Видим, что это число 24. Значит .


Пример 2. Извлечь квадратный корень из числа 432.

Число 432 принадлежит промежутку [100; 10000]. Значит квадратный корень следует искать в промежутке [10; 100]. Открываем таблицу квадратов и смотрим какое число во второй степени равно 432. Обнаруживаем, что число 432 в таблице квадратов отсутствует. В этом случае квадратный корень следует искать приближённо.

Извлечем квадратный корень из числа 432 с точностью до десятых.

В таблице квадратов ближайшее меньшее число к 432 это число 400. Квадратный корень из него равен 20. Отталкиваясь от числа 20, будем проверять различные десятичные дроби, целая часть которых равна 20.

Проверим, например, число 20,8. Для этого возведём его в квадрат:

20,82 = 432,64

Получилось число 432,64 которое превосходит исходное число 432 на 0,64. Видим, что значение 20,8 великó для корня √432. Проверим тогда значение 20,7

20,7= 428,49

Значение 20,7 годится в качестве корня, поскольку в результате возведения этого числа в квадрат получается число 428,49, которое меньше исходного числа 432, но близко к нему. Значит √432 ≈ 20,7.

Необязательно запоминать промежутки чтобы узнать в каких границах располагается корень. Можно воспользоваться методом нахождения ближайших квадратов с чётным количеством нулей на конце.

Например, извлечём корень из числа 4225. Нам известен ближайший меньший квадрат 3600, и ближайший больший квадрат 4900

3600 < 4225 < 4900

Извлечём квадратные корни из чисел 3600 и 4900. Это числа 60 и 70 соответственно:

Тогда можно понять, что квадратный корень из числа 4225 располагается между числами 60 и 70. Можно даже найти его методом подбора. Корни 60 и 70 исключаем сразу, поскольку это корни чисел 3600 и 4900. Затем можно проверить, например, корень 64. Возведём его в квадрат (или умнóжим данное число само на себя)

Корень 64 не годится. Проверим корень 65

Получается 4225. Значит 65 является корнем числа 4225


Тождественные преобразования с квадратными корнями

Над квадратными корнями можно выполнять различные тождественные преобразования, тем самым облегчая их вычисление. Рассмотрим некоторые из этих преобразований.

Квадратный корень из произведения

Квадратный корень из произведения это выражение вида , где a и b некоторые числа.

Например, выражение  является квадратным корнем из произведения чисел 4 и 9.

Чтобы извлечь такой квадратный корень, нужно по отдельности извлечь квадратные корни из множителей 4 и 9, представив выражение  в виде произведения корней . Вычислив по отдельности эти корни полýчим произведение 2 × 3, которое равно 6

Конечно, можно не прибегать к таким манипуляциям, а вычислить сначала подкореннóе выражение 4 × 9, которое равно 36. Затем извлечь квадратный корень из числа 36

Но при извлечении квадратных корней из больших чисел это правило может оказаться весьма полезным.

Допустим, потребовалось извлечь квадратный корень из числа 144. Этот корень легко определяется с помощью таблицы квадратов — он равен 12

Но предстáвим, что таблицы квадратов под рукой не оказалось. В этом случае число 144 можно разложить на простые множители. Затем из этих простых множителей составить числа, квадратные корни из которых извлекаются.

Итак, разлóжим число 144 на простые множители:

Получили следующее разложение:

В разложéнии содержатся четыре двойки и две тройки. При этом все числа, входящие в разложение, перемнóжены. Это позволяет предстáвить произведения одинаковых сомножителей в виде степени с показателем 2.

Тогда четыре двойки можно заменить на запись 2× 22, а две тройки заменить на 32

В результате будем иметь следующее разложение:

Теперь можно извлекáть квадратный корень из разложения числа 144

Применим правило извлечения квадратного корня из произведения:

Ранее было сказано, что если подкореннóе выражение возведенó во вторую степень, то такой квадратный корень равен модулю из подкореннóго выражения.

Тогда получится произведение 2 × 2 × 3, которое равно 12

Простые множители представляют в виде степени для удобства и короткой записи. Допускается также записывать их под кóрнем как есть, чтобы впоследствии перемнóжив их, получить новые сомножители.

Так, разложив число 144 на простые множители, мы получили разложение 2 × 2 × 2 × 2 × 3 × 3. Это разложение можно записать под кóрнем как есть:

затем перемнóжить некоторые сомножители так, чтобы получились числа, квадратные корни из которых извлекаются. В данном случае можно дважды перемнóжить две двойки и один раз перемнóжить две тройки:

Затем применить правило извлечения квадратного корня из произведения и получить окончательный ответ:

С помощью правила извлечения квадратного корня из произведения можно извлекать корень и из других больших чисел. В том числе, из тех чисел, которых нет в таблице квадратов.

Например, извлечём квадратный корень из числа 13456. Этого числа нет в таблице квадратов, поэтому воспользуемся правилом извлечения квадратного корня из произведения, предварительно разложив число 13456 на простые множители.

Итак, разложим число 13456 на простые множители:

В разложении имеются четыре двойки и два числа 29. Двойки дважды предстáвим как 22. А два числа 29 предстáвим как 292. В результате полýчим следующее разложение числа 13456

Теперь будем извлекать квадратный корень из разложения числа 13456

Итак, если ≥ 0 и ≥ 0, то . То есть корень из произведения неотрицательных множителей равен произведению корней из этих множителей.

Докажем равенство . Для этого воспользуемся определением квадратного корня.

Согласно определению, квадратным корня из числа a есть число b, при котором выполняется равенство b= a.

В нашем случае нужно удостовериться, что правая часть равенства  при возведении во вторую степень даст в результате подкореннóе выражение левой части, то есть выражение ab.

Итак, выпишем правую часть равенства  и возведём ее во вторую степень:

Теперь воспользуемся правилом возведения в степень произведения. Согласно этому правилу, каждый множитель данного произведения нужно возвести в указанную степень:

Ранее было сказано, что если выражение вида  возвести во вторую степень, то получится подкореннóе выражение. Применим это правило. Тогда полýчим ab. А это есть подкореннóе выражение квадратного корня

Значит равенство  справедливо, поскольку при возведéнии правой части во вторую степень, получается подкореннóе выражение левой части.

Правило извлечения квадратного корня из произведения работает и в случае, если под кóрнем располагается более двух множителей. То есть справедливым будет следующее равенство:

, при ≥ 0 и ≥ 0, ≥ 0.


Пример 1. Найти значение квадратного корня 

Запишем корень в виде произведения корней, извлечём их, затем найдём значение полученного произведения:


Пример 2. Найти значение квадратного корня 

Предстáвим число 250 в виде произведения чисел 25 и 10. Делать это будем под знáком корня:

Теперь под кóрнем образовалось два одинаковых множителя 10 и 10. Перемнóжим их, полýчим 100

Далее применяем правило извлечения квадратного кóрня из произведения и получáем окончательный ответ:


Пример 3. Найти значение квадратного корня 

Воспользуемся правилом возведения степени в степень. Степень 114 предстáвим как (112)2.

Теперь воспользуемся правилом извлечения квадратного кóрня из квадрата числа:

В нашем случае квадратный корень из числа (112)2 будет равен 112. Говоря простым языком, внешний показатель степени 2 исчезнет, а внутренний останется:

Далее возводим число 11 во вторую степень и получаем окончательный ответ:

Этот пример также можно решить, воспользовавшись правилом извлечения квадратного корня из произведения. Для этого подкореннóе выражение 114 нужно записать в виде произведения 11× 112. Затем извлечь квадратный корень из этого произведения:


Пример 4. Найти значение квадратного корня

Перепишем степень 34 в виде (32)2, а степень 56 в виде (53)2

Далее используем правило извлечения квадратного кóрня из произведения:

Далее используем правило извлечения квадратного кóрня из квадрата числа:

Вычислим произведение получившихся степеней и полýчим окончательный ответ:


Сомножители, находящиеся под корнем, могут быть десятичными дробями. Например, извлечём квадратный корень из произведения

Запишем корень  в виде произведения корней, извлечём их, затем найдём значение полученного произведения:


Пример 6. Найти значение квадратного корня


Пример 7. Найти значение квадратного корня


Если первый сомножитель умножить на число n, а второй сомножитель разделить на это число n, то произведение не изменится.

Например, произведение 8 × 4 равно 32

8 × 4 = 32

Умнóжим сомножитель 8 скажем на число 2, а сомножитель 4 раздéлим на это же число 2. Тогда получится произведение 16 × 2, которое тоже равно 32.

(8 × 2) × (4 : 2) = 32

Это свойство полезно при решении некоторых задач на извлечение квадратных корней. Сомножители подкореннóго выражения можно умнóжить и разделить так, чтобы корни из них извлекались.

Например, извлечём квадратный корень из произведения . Если сразу воспользоваться правилом извлечения квадратного корня из произведения, то не полýчится извлечь корни √1,6 и √90, потому что они не извлекаются.

Проанализировав подкореннóе выражение 1,6 × 90, можно заметить, что если первый сомножитель 1,6 умножить на 10, а второй сомножитель 90 разделить на 10, то полýчится произведение 16 × 9. Из такого произведения квадратный корень можно извлечь, пользуясь правилом извлечения квадратного корня из произведения.

Запишем полное решение данного примера:

Процесс умножения и деления можно выполнять в уме. Также можно пропустить подробную запись извлечения квадратного корня из каждого сомножителя. Тогда решение станóвится короче:


Пример 9. Найти значение квадратного корня

Умнóжим первый сомножитель на 10, а второй раздéлим на 10. Тогда под кóрнем образуется произведение 36 × 0,04, квадратный корень из которого извлекается:


Если в равенстве поменять местами левую и правую часть, то полýчим равенство . Это преобразовáние позволяет упрощáть вычисление некоторых корней.

Например, узнáем чему равно значение выражения .

Квадратные корни из чисел 10 и 40 не извлекаются. Воспользуемся правилом , то есть заменим выражение из двух корней  на выражение с одним корнем, под которым будет произведение из чисел 10 и 40

Теперь найдём значение произведения, находящегося под корнем:

А квадратный корень из числа 400 извлекается. Он равен 20

Сомножители, располагáющиеся под корнем, можно расклáдывать на множители, группировáть, представлять в виде степени, а также перемножáть для получения новых сомножителей, корни из которых извлекаются.

Например, найдём значение выражения .

Воспользуемся правилом

Сомножитель 32 это 25. Предстáвим этот сомножитель как 2 × 24

Перемнóжим сомножители 2 и 2, полýчим 4. А сомножитель 24 предстáвим в виде степени с показателем 2

Теперь воспóльзуемся правилом и вычислим окончательный ответ:


Пример 12. Найти значение выражения

Воспользуемся правилом

Сомножитель 8 это 2 × 2 × 2, а сомножитель 98 это 2 × 7 × 7

Теперь под кóрнем имеются четыре двойки и две семёрки. Четыре двойки можно записать как 2× 22, а две семёрки как 72

Теперь воспользуемся правилом и вычислим окончательный ответ:


Квадратный корень из дроби

Квадратный корень вида равен дроби, в числителе которой квадратный корень из числа a, а в знаменателе — квадратный корень из числа b

Например, квадратный корень из дроби  равен дроби, в числителе которой квадратный корень из числа 4, а в знаменателе — квадратный корень из числа 9

Вычислим квадратные корни в числителе и знаменателе:

Значит, квадратный корень из дроби равен .

Докáжем, что равенство является верным.

Возведём правую часть во вторую степень. Если в результате полýчим дробь , то это будет означать, что равенство верно:


Пример 1. Извлечь квадратный корень 

Воспользуемся правилом извлечения квадратного корня из дроби:


Пример 2. Извлечь квадратный корень 

Переведём подкореннóе выражение в неправильную дробь, затем воспользуемся правилом извлечения квадратного корня из дроби:


Пример 3. Извлечь квадратный корень

Квадратным корнем из числа 0,09 является 0,3. Но можно извлечь этот корень, воспользовавшись правилом извлечения квадратного корня из дроби.

Предстáвим подкоренное выражение в виде обыкновенной дроби. 0,09 это девять сотых:

Теперь можно воспользоваться правилом извлечения квадратного корня из дроби:


Пример 4. Найти значение выражения 

Извлечём корни из 0,09 и 0,25, затем сложим полученные результаты:

Также можно воспользоваться правилом извлечения квадратного корня из дроби:

В данном примере первый способ оказался проще и удобнее.


Пример 5. Найти значение выражения 

Сначала вычислим квадратный корень, затем перемнóжим его с 10. Получившийся результат вычтем из 4


Пример 6. Найти значение выражения 

Сначала найдём значение квадратного корня . Он равен 0,6 поскольку 0,6= 0,36

Теперь вычислим получившееся выражение. Согласно порядку действий, сначала надо выполнить умножение, затем сложение:


Вынесение множителя из-под знака корня

В некоторых задачах может быть полезным вынесение множителя из-под знака корня.

Рассмотрим квадратный корень из произведения . Согласно правилу извлечения квадратного корня из произведения, нужно извлечь квадратный корень из каждого множителя данного произведения:

В нашем примере квадратный корень извлекается только из множителя 4. Его мы извлечём, а выражение  оставим без изменений:

Это и есть вынесение множителя из-под знака корня.

На практике подкореннóе выражение чаще всего требуется разложить на множители.


Пример 2. Вынести множитель из-под знака корня в выражении

Разлóжим подкореннóе выражение на множители 9 и 2. Тогда полýчим:

Теперь воспользуемся правило извлечения квадратного корня из произведения. Извлечь можно только корень из множителя 9. Множитель 2 остáвим под кóрнем:


Пример 3. Вынести множитель из-под знака корня в выражении

Разлóжим подкореннóе выражение на множители 121 и 3. Тогда полýчим:

Теперь воспользуемся правилом извлечения квадратного корня из произведения. Извлечь можно только корень из множителя 121. Выражение √3 остáвим под корнем:


Пример 4. Вынести множитель из-под знака корня в выражении

Воспользуемся правилом извлечения квадратного корня из произведения:

Квадратный корень извлекается только из числа 121. Извлечём его, а выражение √15 оставим без изменений:

Получается, что множитель 11 вынесен из-под знака корня. Вынесенный множитель принято записывать до выражения с корнем. Поменяем выражения √15 и 11 местами:


Пример 5. Вынести множитель из-под знака корня в выражении

Разлóжим подкореннóе выражение на множители 4 и 3

Воспользуемся правилом извлечения квадратного корня из произведения:

Извлечём корень из числа 4, а выражение √3 остáвим без изменений:


Пример 6. Упростить выражение 

Предстáвим второе слагаемое в виде . А третье слагаемое предстáвим в виде

Теперь в выражениях и вынесем множитель из-под знака корня:

Во втором слагаемом перемнóжим числа −4 и 4. Остальное перепишем без изменений:

Замечáем, что получившемся выражении квадратный корень √3 является общим множителем. Вынесем его за скобки:

Вычислим содержимое скобок, полýчим −1

Если множителем является −1, то записывают только минус. Единица опускается. Тогда полýчим окончательный ответ −√3


Внесение множителя под знак корня

Рассмотрим следующее выражение:

В этом выражении число 5 умнóжено на квадратный корень из числа 9. Найдём значение этого выражения.

Сначала извлечём квадратный корень, затем перемнóжим его с числом 5.

Квадратный корень из 9 равен 3. Перемнóжим его с числом 5. Тогда полýчим 15

Число 5 в данном случае было множителем. Внесём этот множитель под знак корня. Но сделать это нужно таким образом, чтобы в результате наших действий значение исходного выражения не изменилось. Проще говоря, после внесения множителя 5 под знак корня, получившееся выражение по-прежнему должно быть равно 15.

Значение выражения не изменится, если число 5 возвести во вторую степень и только тогда внести его под корень:

Итак, если данó выражение , и нужно внести множитель a под знак корня, то надо возвести во вторую степень множитель a и внести его под корень:

Пример 1. Внести множитель под знак корня в выражении

Возведём число 7 во вторую степень и внесём его под знак корня:


Пример 2. Внести множитель под знак корня в выражении 

Возведём число 10 во вторую степень и внесем его под знак корня:


Пример 3. Внести множитель под знак корня в выражении 

Вносить под знак корня можно только положительный множитель. Ранее было сказано, что выражение вида  не имеет смысла.

Однако, если перед знаком кóрня располагается отрицательный множитель, то минус можно оставить за знáком корня, а самó число внести под знак корня.

Пример 4. Внести множитель по знак корня в выражении 

В этом примере под знак корня внóсится только 3. Минус остаётся за знáком корня:


Пример 5. Выполнить возведéние в степень в следующем выражении:

Воспользуемся формулой квадрата суммы двух выражений:

(a + b)2 = a+ 2ab + b2

Роль переменной a в данном случае играет выражение √3, роль переменной b — выражение √2. Тогда полýчим:

Теперь необходимо упростить получившееся выражение.

Для выражений и  применим правило . Ранее мы говорили, что если выражение вида  возвести во вторую степень, то это выражение будет равно подкореннóму выражению a.

А в выражении для множителей и применим правило . То есть заменим произведение корней на один общий корень:

Приведём подобные слагаемые. В данном случае можно сложить слагаемые 3 и 2. А в слагаемом вычислить произведение, которое под кóрнем:


 

Задания для самостоятельного решения

Задание 1. Найдите значение квадратного корня:

Решение:

Задание 2. Найдите значение квадратного корня:

Решение:

Задание 3. Найдите значение квадратного корня:

Решение:

Задание 4. Найдите значение выражения:

Решение:

Задание 5. Найдите значение квадратного корня:

Решение:

Задание 6. Найдите значение квадратного корня:

Решение:

Задание 7. Найдите значение квадратного корня:

Решение:

Задание 8. Найдите значения следующих выражений:

Решение:

Задание 9. Извлеките квадратный корень из числа 4624

Решение:

Задание 10. Извлеките квадратный корень из числа 11025

Решение:

Задание 11. Найдите значение квадратного корня:

Решение:

Задание 12. Найдите значение квадратного корня:

Решение:

Задание 13. Найдите значение квадратного корня:

Решение:

Задание 14. Найдите значение квадратного корня:

Решение:

Задание 15. Найдите значение квадратного корня:

Решение:

Задание 16. Найдите значение выражения:

Решение:

Задание 17. Найдите значение выражения:

Решение:

Задание 18. Найдите значение выражения:

Решение:

Задание 19. Найдите значение выражения:

Решение:

Задание 20. Найдите значение выражения:

Решение:

Задание 21. Найдите значение выражения:

Решение:

Задание 22. Найдите значение выражения:

Решение:

Задание 23. Найдите значение выражения:

Решение:

Задание 24. Найдите значение выражения:

Решение:

Задание 25. Найдите значение выражения:

Решение:

Задание 26. Найдите значение выражения:

Решение:

Задание 27. Найдите значение выражения:

Решение:

Задание 28. Найдите значение выражения:

Решение:

Задание 29. Найдите значение выражения:

Решение:

Задание 30. Найдите значение выражения:

Решение:

Задание 31. Найдите значение выражения:

Решение:

Задание 32. Найдите значение выражения:

Решение:

Задание 33. Найдите значение выражения:

Решение:

Задание 34. Вынести множитель из-под знака корня:

Решение:

Задание 35. Вынести множитель из-под знака корня:

Решение:

Задание 36. Вынести множитель из-под знака корня:

Решение:

Задание 37. Вынести множитель из-под знака корня:

Решение:

Задание 38. Вынести множитель из-под знака корня:

Решение:

Задание 39. Вынести множитель из-под знака корня:

Решение:

Задание 40. Вынести множитель из-под знака корня:

Решение:

Задание 41. Вынести множитель из-под знака корня:

Решение:

Задание 42. Вынести множитель из-под знака корня:

Решение:

Задание 43. Вынести множитель из-под знака корня:

Решение:

Задание 44. Вынести множитель из-под знака корня в следующих выражениях:

Решение:

Задание 45. Внести множитель под знак корня:

Решение:

Задание 46. Внести множитель под знак корня:

Решение:

Задание 47. Внести множитель под знак корня:

Решение:

Задание 48. Внести множитель под знак корня:

Решение:

Задание 49. Внести множитель под знак корня:

Решение:

Задание 50. Внести множитель под знак корня в следующих выражениях:

Решение:

Задание 51. Упростить выражение:

Решение:

Задание 52. Упростить выражение:

Решение:

Задание 53. Упростить выражение:

Решение:

Задание 54. Упростить выражение:

Решение:

Задание 55. Упростить выражение:

Решение:

Задание 56. Упростить выражение:

Решение:

Задание 57. Упростить выражение:

Решение:

Задание 58. Упростить выражение:

Решение:

Задание 59. Упростить выражение:

Решение:

Задание 60. Упростить выражение:

Решение:


Понравился урок?
Вступай в нашу новую группу Вконтакте и начни получать уведомления о новых уроках

Возникло желание поддержать проект?
Используй кнопку ниже

Навигация по записям

Функция КОРЕНЬ

В этой статье описаны синтаксис формулы и использование функции КОРЕНЬ в Microsoft Excel.

Описание

Возвращает положительное значение квадратного корня.

Синтаксис

КОРЕНЬ(число)

Аргументы функции КОРЕНЬ описаны ниже.

Замечание

Если число отрицательное, то SQRT возвращает #NUM! значение ошибки #ЗНАЧ!.

Пример

Скопируйте образец данных из следующей таблицы и вставьте их в ячейку A1 нового листа Excel. Чтобы отобразить результаты формул, выделите их и нажмите клавишу F2, а затем — клавишу ВВОД. При необходимости измените ширину столбцов, чтобы видеть все данные.

Данные

-16

Формула

Описание

Результат

=КОРЕНЬ(16)

Квадратный корень числа 16.

4

=КОРЕНЬ(A2)

Квадратный корень -16. Так как число отрицательное, #NUM! возвращается сообщение об ошибке.

#ЧИСЛО!

=КОРЕНЬ(ABS(A2))

Старайтесь не #NUM! сообщение об ошибке: сначала с помощью функции ABS можно найти абсолютное значение -16, а затем найти квадратный корень.

4

Счет, степени, корни — Математика — Теория, тесты, формулы и задачи

Оглавление:

 

Основные теоретические сведения

Некоторые рекомендации к проведению алгебраических вычислений, преобразований и упрощений

К оглавлению. ..

При выполнении численных вычислений с большим количеством операций и дробей желательно выполнять следующие рекомендации:

  • Переводите десятичные дроби в обыкновенные, т.е. такие у которых есть числитель и знаменатель.
  • Не старайтесь посчитать сразу все выражение. Выполняйте вычисления по одному действию, пошагово. При этом учтите, что:
    • сначала выполняют операции в скобках;
    • затем считают произведения и/или деления;
    • потом суммируют или вычитают;
    • и в последнюю очередь, если это была многоэтажная дробь, делят уже полностью упрощенный числитель на тоже полностью упрощенный знаменатель;
    • причем выполняя в первую очередь операции в скобках также соблюдают ту же последовательность, сначала произведения или деления внутри скобок, потом суммирование или вычитание в скобках, а если внутри скобки есть другая скобка то действия в ней выполняются прежде всего.
  • Не спешите умножать и делить «страшные числа».  Скорее всего, в одном из следующих действий что-то сократится. Чтобы проще было сократить можно числа раскладывать на простые множители.
  • При сложении и вычитании выделяйте в дробях целую часть (если это возможно). При умножении и делении, наоборот, приводите дробь к виду без целой части.

От корней в знаменателе принято избавляться. Для избавления от корня над всем знаменателем умножают числитель и знаменатель на выражение, равное знаменателю. Для избавления от корня над частью знаменателя умножают числитель и знаменатель на сопряженное знаменателю выражение. В этом случае образуется разность квадратов (сопряжённым для (ab) является выражение (a + b) и наоборот).

При преобразовании или упрощении алгебраических выражений последовательность действий такова:

  • Разложить на множители все, что можно разложить на множители.
  • Сократить все, что можно сократить.
  • И только потом приводить к общему знаменателю. Ни в коем случае не пытайтесь сразу сломя голову приводить к общему знаменателю. Пример будет становиться чем дальше, тем страшнее.
  • Снова разложить на множители и сократить.

Для того чтобы перевести десятичную периодическую дробь в обыкновенную (с числителем и знаменателем) необходимо:

  • Из числа, стоящего до второго периода в исходной периодической дроби вычесть число, стоящее до первого периода в этой же дроби и записать полученную разность в числитель будущей обыкновенной дроби.
  • В знаменателе же записать столько девяток, сколько цифр в периоде исходной дроби, и столько нулей, сколько цифр между запятой и первым периодом.
  • Не забыть про целую часть, если она есть.

При решении задач из данной темы также необходимо помнить много сведений из предыдущих тем. Приведём далее основные из них.

 

Формулы сокращенного умножения

К оглавлению. ..

При выполнении различных алгебраических преобразований часто удобно пользоваться формулами сокращенного умножения. Зачастую эти формулы применяются не столько для того чтобы сократить процесс умножения, а наоборот скорее для того, чтобы по результату понять, что его можно представить как произведение некоторых множителей. Таким образом, данные формулы нужно уметь применять не только слева направо, но и справа налево. Перечислим основные формулы сокращенного умножения:

Последние две формулы также часто удобно использовать в виде:

 

Квадратный трехчлен и теорема Виета

К оглавлению…

В случае когда квадратное уравнение имеет два корня, соответствующий квадратный трехчлен может быть разложен на множители по следующей формуле:

Если квадратное уравнение имеет один корень, то разложение соответствующего квадратного трехчлена на множители задается следующей формулой:

Только в случае если квадратное уравнение имеет два корня (т. е. дискриминант строго больше ноля) выполняется Теорема Виета. Согласно Теореме Виета, сумма корней квадратного уравнения равна:

Произведение корней квадратного уравнения согласно теореме Виета может быть вычислено по формуле:

Итак, еще раз о теореме Виета:

  • Если D < 0 (дискриминант отрицателен), то уравнение корней не имеет и теорему Виета применять нельзя.
  • Если D > 0 (дискриминант положителен), то уравнение имеет два корня и теорема Виета прекрасно работает.
  • Если D = 0, то уравнение имеет единственный корень, для которого бессмысленно вводить понятие суммы или произведения корней, поэтому теорему Виета тоже не применяем.

 

Основные свойства степеней

К оглавлению…

У математических степеней есть несколько важных свойств, перечислим их:

Последнее свойство выполняется только при n > 0. Ноль можно возводить только в положительную степень. Ну а основное свойство отрицательной степени записывается следующим образом:

 

Основные свойства математических корней

К оглавлению…

Математический корень можно представить в виде обычной степени, а затем пользоваться всеми свойствами степеней приведёнными выше. Для представления математического корня в виде степени используют следующую формулу:

Тем не менее можно отдельно выписать ряд свойств математических корней, которые основываются на свойствах степеней описанных выше:

Для арифметических корней выполняется следующее свойство (которое одновременно можно считать определением корня):

Последнее справедливо: если n – нечетное, то для любого a; если же n – четное, то только при a больше либо равном нолю. Для корня нечетной степени выполняется также следующее равенство (из под корня нечетной степени можно выносить знак «минус»):

Так как значение корня четной степени может быть только неотрицательным, то для таких корней имеется следующее важное свойство:

Итак всегда нужно помнить, что под корнем четной степени может стоять только неотрицательное выражение, и сам корень тоже есть неотрицательное выражение.  Кроме того, нужно отметить, что если используется запись со значком математического корня, то показатель степени этого корня может быть только целым числом, причем это число должно быть больше либо равно двум:

 

Основные свойства квадратного корня

К оглавлению…

Квадратным корнем называется математический корень второй степени:

Квадратный корень можно извлечь только из неотрицательного числа. При этом значение квадратного корня также всегда неотрицательно:

Для квадратного корня существует два важных свойства, которые важно очень хорошо запомнить и не путать:

Если под корнем стоит несколько множителей, то корень можно извлекать из каждого из них по-отдельности. При этом важно понимать, что каждый из этих множителей по-отдельности (а не только их произведение) должны быть неотрицательными:

Что такое квадратный корень определение. Формулы корней

Рациональные числа

Неотрицательный квадратный корень из положительного числа называется арифметическим квадратным корнем и обозначается с использованием знака радикала .

Комплексные числа

Над полем комплексных чисел решений всегда два, отличающихся только знаком (за исключением квадратного корня из нуля). Корень из комплексного числа часто обозначают как , однако использовать это обозначение нужно осторожно. Распространённая ошибка:

Для извлечения квадратного корня из комплексного числа удобно использовать экспоненциальную форму записи комплексного числа: если

, ,

где корень из модуля понимается в смысле арифметического значения, а k может принимать значения k=0 и k=1, таким образом в итоге в ответе получаются два различных результата.


Обобщения

Квадратные корни вводятся как решения уравнений вида и для других объектов: матриц , функций , операторов и т. п. В качестве операции при этом могут использоваться достаточно произвольные мультипликативные операции, например, суперпозиция .

Квадратный корень в информатике

Во многих языках программирования функционального уровня (а также языках разметки типа LaTeX) функция квадратного корня обозначается как sqrt (от англ. square root «квадратный корень»).

Алгоритмы нахождения квадратного корня

Нахождение или вычисление квадратного корня заданного числа называется извлечением (квадратного) корня.

Разложение в ряд Тейлора

при .

Арифметическое извлечение квадратного корня

Для квадратов чисел верны следующие равенства:

То есть, узнать целую часть квадратного корня числа можно, вычитая из него все нечётные числа по порядку, пока остаток не станет меньше следующего вычитаемого числа или равен нулю, и посчитав количество выполненных действий. Например, так:

Выполнено 3 действия, квадратный корень числа 9 равен 3.

Недостатком такого способа является то, что если извлекаемый корень не является целым числом, то можно узнать только его целую часть, но не точнее. В то же время такой способ вполне доступен детям, решающим простейшие математические задачи, требующие извлечения квадратного корня.

Грубая оценка

Многие алгоритмы вычисления квадратных корней из положительного действительного числа S требуют некоторого начального значения. Если начальное значение слишком далеко от настоящего значения корня, вычисления замедляются. Поэтому полезно иметь грубую оценку, которая может быть очень неточна, но легко вычисляется. Если S ≥ 1, пусть D будет числом цифр S слева от десятичной запятой. Если S D будет числом нулей, идущих подряд, справа от десятичной запятой, взятое со знаком минус. Тогда грубая оценка выглядит так:

Если D нечётно, D = 2n + 1, тогда используем Если D чётно, D = 2n + 2, тогда используем

Два и шесть используются потому, что и

При работе в двоичной системе (как внутри компьютеров), следует использовать другую оценку (здесь D это число двоичных цифр).

Геометрическое извлечение квадратного корня

Для ручного извлечения корня применяется запись, похожая на деление столбиком . Выписывается число, корень которого ищем. Справа от него будем постепенно получать цифры искомого корня. Пусть извлекается корень из числа с конечным числом знаков после запятой. Для начала мысленно или метками разобьём число N на группы по две цифры слева и справа от десятичной точки. При необходимости, группы дополняются нулями — целая часть дополняется слева, дробная справа. Так 31234.567 можно представить, как 03 12 34 . 56 70. В отличие от деления снос производится такими группами по 2 цифры.

Наглядное описание алгоритма:

Площадь квадратного участка земли равна 81 дм². Найти его сторону. Предположим, что длина стороны квадрата равна х дециметрам. Тогда площадь участка равна х ² квадратным дециметрам. Так как по условию эта площадь равна 81 дм², то х ² = 81. Длина стороны квадрата — положительное число. Положительным числом, квадрат которого равен 81, является число 9. При решении задачи требовалось найти число х, квадрат которого равен 81, т. е. решить уравнение х ² = 81. Это уравнение имеет два корня: x 1 = 9 и x 2 = — 9, так как 9² = 81 и (- 9)² = 81. Оба числа 9 и — 9 называют квадратными корнями из числа 81.

Заметим, что один из квадратных корней х = 9 является положительным числом. Его называют арифметическим квадратным корнем из числа 81 и обозначают √81, таким образом √81 = 9.

Арифметическим квадратным корнем из числа а называется неотрицательное число, квадрат которого равен а .

Например, числа 6 и — 6 являются квадратными корнями из числа 36. При этом число 6 является арифметическим квадратным корнем из 36, так как 6 — неотрицательное число и 6² = 36. Число — 6 не является арифметическим корнем.

Арифметический квадратный корень из числа а обозначается так: √а.

Знак называется знаком арифметического квадратного корня; а — называется подкоренным выражением. Выражение √а читается так: арифметический квадратный корень из числа а. Например, √36 = 6, √0 = 0, √0,49 = 0,7. В тех случаях, когда ясно, что речь идет об арифметическом корне, кратко говорят: «корень квадратный из а «.

Действие нахождения квадратного корня из числа называют извлечением квадратного корня. Это действие является обратным к возведению в квадрат.

Возводить в квадрат можно любые числа, но извлекать квадратные корни можно не из любого числа. Например, нельзя извлечь квадратный корень из числа — 4. Если бы такой корень существовал, то, обозначив его буквой х , мы получили бы неверное равенство х² = — 4, так как слева стоит неотрицательное число, а справа отрицательное.

Выражение √а имеет смысл только при а ≥ 0. Определение квадратного корня можно кратко записать так: √а ≥ 0, (√а )² = а . Равенство (√а )² = а справедливо при а ≥ 0. Таким образом, чтобы убедиться в том, что квадратный корень из неотрицательного числа а равен b , т. е. в том, что √а =b , нужно проверить, что выполняются следующие два условия: b ≥ 0, b ² = а.

Квадратный корень из дроби

Вычислим . Заметим, что √25 = 5, √36 = 6, и проверим выполняется ли равенство .

Так как и , то равенство верно. Итак, .

Теорема: Если а ≥ 0 и b > 0, то т. е. корень из дроби равен корню из числителя, деленному на корень из знаменателя. Требуется доказать, что: и .

Так как √а ≥0 и √b > 0, то .

По свойству возведения дроби в степень и определению квадратного корня теорема доказана. Рассмотрим несколько примеров.

Вычислить , по доказанной теореме .

Второй пример: Доказать, что , если а ≤ 0, b .

Еще примерчик: Вычислить .

.

Преобразование квадратных корней

Вынесение множителя из-под знака корня. Пусть дано выражение . Если а ≥ 0 и b ≥ 0, то по теореме о корне из произведения можно записать:

Такое преобразование называется вынесение множителя из под знака корня. Рассмотрим пример;

Вычислить при х = 2. Непосредственная подстановка х = 2 в подкоренное выражение приводит к сложным вычислениям. 2=400\\ \hline \end{array}\]

Факт 3.
Какие действия можно выполнять с квадратными корнями?
\(\bullet\) Сумма или разность квадратных корней НЕ РАВНА квадратному корню из суммы или разности, то есть \[\sqrt a\pm\sqrt b\ne \sqrt{a\pm b}\] Таким образом, если вам нужно вычислить, например, \(\sqrt{25}+\sqrt{49}\) , то первоначально вы должны найти значения \(\sqrt{25}\) и \(\sqrt{49}\) , а затем их сложить. Следовательно, \[\sqrt{25}+\sqrt{49}=5+7=12\] Если значения \(\sqrt a\) или \(\sqrt b\) при сложении \(\sqrt a+\sqrt b\) найти не удается, то такое выражение дальше не преобразуется и остается таким, как есть. Например, в сумме \(\sqrt 2+ \sqrt {49}\) мы можем найти \(\sqrt{49}\) – это \(7\) , а вот \(\sqrt 2\) никак преобразовать нельзя, поэтому \(\sqrt 2+\sqrt{49}=\sqrt 2+7\) . Дальше это выражение, к сожалению, упростить никак нельзя \(\bullet\) Произведение/частное квадратных корней равно квадратному корню из произведения/частного, то есть \[\sqrt a\cdot \sqrt b=\sqrt{ab}\quad \text{и}\quad \sqrt a:\sqrt b=\sqrt{a:b}\] (при условии, что обе части равенств имеют смысл )
Пример: \(\sqrt{32}\cdot \sqrt 2=\sqrt{32\cdot 2}=\sqrt{64}=8\) ; \(\sqrt{768}:\sqrt3=\sqrt{768:3}=\sqrt{256}=16\) ; \(\sqrt{(-25)\cdot (-64)}=\sqrt{25\cdot 64}=\sqrt{25}\cdot \sqrt{64}= 5\cdot 8=40\) . \(\bullet\) Пользуясь этими свойствами, удобно находить квадратные корни из больших чисел путем разложения их на множители.
Рассмотрим пример. Найдем \(\sqrt{44100}\) . Так как \(44100:100=441\) , то \(44100=100\cdot 441\) . По признаку делимости число \(441\) делится на \(9\) (так как сумма его цифр равна 9 и делится на 9), следовательно, \(441:9=49\) , то есть \(441=9\cdot 49\) .
Таким образом, мы получили: \[\sqrt{44100}=\sqrt{9\cdot 49\cdot 100}= \sqrt9\cdot \sqrt{49}\cdot \sqrt{100}=3\cdot 7\cdot 10=210\] Рассмотрим еще один пример: \[\sqrt{\dfrac{32\cdot 294}{27}}= \sqrt{\dfrac{16\cdot 2\cdot 3\cdot 49\cdot 2}{9\cdot 3}}= \sqrt{ \dfrac{16\cdot4\cdot49}{9}}=\dfrac{\sqrt{16}\cdot \sqrt4 \cdot \sqrt{49}}{\sqrt9}=\dfrac{4\cdot 2\cdot 7}3=\dfrac{56}3\]
\(\bullet\) Покажем, как вносить числа под знак квадратного корня на примере выражения \(5\sqrt2\) (сокращенная запись от выражения \(5\cdot \sqrt2\) ). Так как \(5=\sqrt{25}\) , то \ Заметим также, что, например,
1) \(\sqrt2+3\sqrt2=4\sqrt2\) ,
2) \(5\sqrt3-\sqrt3=4\sqrt3\)
3) \(\sqrt a+\sqrt a=2\sqrt a\) . 2\) , поэтому \(\sqrt{16}=4\) . А вот извлечь корень из числа \(3\) , то есть найти \(\sqrt3\) , нельзя, потому что нет такого числа, которое в квадрате даст \(3\) .
Такие числа (или выражения с такими числами) являются иррациональными. Например, числа \(\sqrt3, \ 1+\sqrt2, \ \sqrt{15}\) и т.п. являются иррациональными.
Также иррациональными являются числа \(\pi\) (число “пи”, приблизительно равное \(3,14\) ), \(e\) (это число называют числом Эйлера, приблизительно оно равно \(2,7\) ) и т.д.
\(\bullet\) Обращаем ваше внимание на то, что любое число будет либо рациональным, либо иррациональным. А вместе все рациональные и все иррациональные числа образуют множество, называющееся множеством действительных (вещественных) чисел. Обозначается это множество буквой \(\mathbb{R}\) .
Значит, все числа, которые на данный момент мы знаем, называются вещественными числами.

Факт 5.
\(\bullet\) Модуль вещественного числа \(a\) – это неотрицательное число \(|a|\) , равное расстоянию от точки \(a\) до \(0\) на вещественной прямой. 2\\ &2>2,25 \end{aligned}\] Видим, что мы получили неверное неравенство. Следовательно, наше предположение было неверным и \(\sqrt 2-1Заметим, что прибавление некоторого числа к обеим частям неравенства не влияет на его знак. Умножение/деление обеих частей неравенства на положительное число также не влияет на его знак, а умножение/деление на отрицательное число меняет знак неравенства на противоположный!
Возводить обе части уравнения/неравенства в квадрат можно ТОЛЬКО ТОГДА, когда обе части неотрицательные. Например, в неравенстве из предыдущего примера возводить обе части в квадрат можно, в неравенстве \(-3 \(\bullet\) Следует запомнить, что \[\begin{aligned} &\sqrt 2\approx 1,4\\ &\sqrt 3\approx 1,7 \end{aligned}\] Знание приблизительного значения данных чисел поможет вам при сравнении чисел! \(\bullet\) Для того, чтобы извлечь корень (если он извлекается) из какого-то большого числа, которого нет в таблице квадратов, нужно сначала определить, между какими “сотнями” оно находится, затем – между какими “десятками”, а потом уже определить последнюю цифру этого числа. 2=168\cdot 168=28224\) .
Следовательно, \(\sqrt{28224}=168\) . Вуаля!

Для того чтобы достойно решить ЕГЭ по математике, прежде всего необходимо изучить теоретический материал, который знакомит с многочисленными теоремами, формулами, алгоритмами и т. д. На первый взгляд может показаться, что это довольно просто. Однако найти источник, в котором теория для ЕГЭ по математике изложена легко и понятно для учащихся с любым уровнем подготовки, — на деле задача довольно сложная. Школьные учебники невозможно всегда держать под рукой. А найти основные формулы для ЕГЭ по математике бывает непросто даже в Интернете.

Почему так важно изучать теорию по математике не только для тех, кто сдает ЕГЭ?

  1. Потому что это расширяет кругозор . Изучение теоретического материала по математике полезно для всех, кто желает получить ответы на широкий круг вопросов, связанных с познанием окружающего мира. Все в природе упорядоченно и имеет четкую логику. Именно это и отражается в науке, через которую возможно понять мир.
  2. Потому что это развивает интеллект . Изучая справочные материалы для ЕГЭ по математике, а также решая разнообразные задачи, человек учится логически мыслить и рассуждать, грамотно и четко формулировать мысли. У него вырабатывается способность анализировать, обобщать, делать выводы.

Предлагаем вам лично оценить все преимущества нашего подхода к систематизации и изложению учебных материалов.

Математика зародилась тогда, когда человек осознал себя и стал позиционироваться как автономная единица мира. Желание измерить, сравнить, посчитать то, что тебя окружает, — вот что лежало в основе одной из фундаментальных наук наших дней. Сначала это были частички элементарной математики, что позволили связать числа с их физическими выражениями, позже выводы стали излагаться лишь теоретически (в силу своей абстрактности), ну а через некоторое время, как выразился один ученый, «математика достигла потолка сложности, когда из нее исчезли все числа». Понятие «квадратный корень» появилось еще в то время, когда его можно было без проблем подкрепить эмпирическими данными, выходя за плоскость вычислений.

С чего все начиналось

Первое упоминание корня, который на данный момент обозначается как √, было зафиксировано в трудах вавилонских математиков, положивших начало современной арифметике. Конечно, на нынешнюю форму они походили мало — ученые тех лет сначала пользовались громоздкими табличками. Но во втором тысячелетии до н. э. ими была выведена приближенная формула вычислений, которая показывала, как извлечь квадратный корень. На фото ниже изображен камень, на котором вавилонские ученые высекли процесс вывода √2 , причем он оказался настолько верным, что расхождение в ответе нашли лишь в десятом знаке после запятой.

Помимо этого, корень применялся, если нужно было найти сторону треугольника, при условии, что две другие известны. Ну и при решении квадратных уравнений от извлечения корня никуда не деться.

Наравне с вавилонскими работами объект статьи изучался и в китайской работе «Математика в девяти книгах», а древние греки пришли к выводу, что любое число, из которого не извлекается корень без остатка, дает иррациональный результат.

Происхождение данного термина связывают с арабским представлением числа: древние ученые полагали, что квадрат произвольного числа произрастает из корня, подобно растению. На латыни это слово звучит как radix (можно проследить закономерность — все, что имеет под собой «корневую» смысловую нагрузку, созвучно, будь то редис или радикулит).

Ученые последующих поколений подхватили эту мысль, обозначая его как Rx. Например, в XV веке, дабы указать, что извлекается корень квадратный из произвольного числа a, писали R 2 a. Привычная современному взгляду «галочка» √ появилась лишь в XVII веке благодаря Рене Декарту.

Наши дни

С точки зрения математики, квадратный корень из числа y — это такое число z, квадрат которого равен y. Иными словами, z 2 =y равносильно √y=z. Однако данное определение актуально лишь для арифметического корня, так как оно подразумевает неотрицательное значение выражения. Иными словами, √y=z, где z больше либо равно 0.

В общем случае, что действует для определения алгебраического корня, значение выражения может быть как положительным, так и отрицательным. Таким образом, в силу того, что z 2 =y и (-z) 2 =y, имеем: √y=±z или √y=|z|.

Благодаря тому, что любовь к математике с развитием науки лишь возросла, существуют разнообразные проявления привязанности к ней, не выраженные в сухих вычислениях. Например, наравне с такими занятными явлениями, как день числа Пи, отмечаются и праздники корня квадратного. Отмечаются они девять раз в сто лет, и определяются по следующему принципу: числа, которые обозначают по порядку день и месяц, должна быть корнем квадратным из года. Так, в следующий раз предстоит отмечать сей праздник 4 апреля 2016 года.

Свойства квадратного корня на поле R

Практически все математические выражения имеют под собой геометрическую основу, не миновала эта участь и √y, который определяется как сторона квадрата с площадью y.

Как найти корень числа?

Алгоритмов вычисления существует несколько. Наиболее простым, но при этом достаточно громоздким, является обычный арифметический подсчет, который заключается в следующем:

1) из числа, корень которого нам нужен, по очереди вычитаются нечетные числа — до тех пор, пока остаток на выходе не получится меньше вычитаемого или вообще будет равен нулю. Количество ходов и станет в итоге искомым числом. Например, вычисление квадратного корня из 25:

Следующее нечетное число — это 11, остаток у нас следующий: 1

Для таких случаев существует разложение в ряд Тейлора:

√(1+y)=∑((-1) n (2n)!/(1-2n)(n!) 2 (4 n))y n , где n принимает значения от 0 до

+∞, а |y|≤1.

Графическое изображение функции z=√y

Рассмотрим элементарную функцию z=√y на поле вещественных чисел R, где y больше либо равен нулю. График ее выглядит следующим образом:

Кривая растет из начала координат и обязательно пересекает точку (1; 1).

Свойства функции z=√y на поле действительных чисел R

1. Область определения рассматриваемой функции — промежуток от нуля до плюс бесконечности (ноль включен).

2. Область значений рассматриваемой функции — промежуток от нуля до плюс бесконечности (ноль опять же включен).

3. Минимальное значение (0) функция принимает лишь в точке (0; 0). Максимальное значение отсутствует.

4. Функция z=√y ни четная, ни нечетная.

5. Функция z=√y не является периодической.

6. Точка пересечения графика функции z=√y с осями координат лишь одна: (0; 0).

7. Точка пересечения графика функции z=√y также является и нулем этой функции.

8. Функция z=√y непрерывно растет.

9. Функция z=√y принимает лишь положительные значения, следовательно, график ее занимает первый координатный угол.

Варианты изображения функции z=√y

В математике для облегчения вычислений сложных выражений порой используют степенную форму написания корня квадратного: √y=y 1/2 . Такой вариант удобен, например, в возведении функции в степень: (√y) 4 =(y 1/2) 4 =y 2 . Этот метод является удачным представлением и при дифференцировании с интегрированием, так как благодаря ему корень квадратный представляется обычной степенной функцией.

А в программировании заменой символа √ является комбинация букв sqrt.

Стоит отметить, что в данной области квадратный корень очень востребован, так как входит в состав большинства геометрических формул, необходимых для вычислений. Сам алгоритм подсчета достаточно сложен и строится на рекурсии (функции, что вызывает сама себя).

Корень квадратный в комплексном поле С

По большому счету именно предмет данной статьи стимулировал открытие поля комплексных чисел C, так как математикам не давал покоя вопрос получения корня четной степени из отрицательного числа. Так появилась мнимая единица i, которая характеризуется очень интересным свойством: ее квадратом есть -1. Благодаря этому квадратные уравнения и при отрицательном дискриминанте получили решение. В С для корня квадратного актуальны те же свойства, что и в R, единственное, сняты ограничения с подкоренного выражения.

До появления калькуляторов студенты и преподаватели вычисляли квадратные корни вручную. Существует несколько способов вычисления квадратного корня числа вручную. Некоторые из них предлагают только приблизительное решение, другие дают точный ответ.

Шаги

Разложение на простые множители

    Разложите подкоренное число на множители, которые являются квадратными числами. В зависимости от подкоренного числа, вы получите приблизительный или точный ответ. Квадратные числа – числа, из которых можно извлечь целый квадратный корень. Множители – числа, которые при перемножении дают исходное число. Например, множителями числа 8 являются 2 и 4, так как 2 х 4 = 8, числа 25, 36, 49 являются квадратными числами, так как √25 = 5, √36 = 6, √49 = 7. Квадратные множители – это множители, которые являются квадратными числами. Сначала попытайтесь разложить подкоренное число на квадратные множители.

  • Например, вычислите квадратный корень из 400 (вручную). Сначала попытайтесь разложить 400 на квадратные множители. 400 кратно 100, то есть делится на 25 – это квадратное число. Разделив 400 на 25, вы получите 16. Число 16 также является квадратным числом. Таким образом, 400 можно разложить на квадратные множители 25 и 16, то есть 25 х 16 = 400.
  • Записать это можно следующим образом: √400 = √(25 х 16).
  • Квадратные корень из произведения некоторых членов равен произведению квадратных корней из каждого члена, то есть √(а х b) = √a x √b. Воспользуйтесь этим правилом и извлеките квадратный корень из каждого квадратного множителя и перемножьте полученные результаты, чтобы найти ответ.

    • В нашем примере извлеките корень из 25 и из 16.
      • √(25 х 16)
      • √25 х √16
      • 5 х 4 = 20
  • Если подкоренное число не раскладывается на два квадратных множителя (а так происходит в большинстве случаев), вы не сможете найти точный ответ в виде целого числа. Но вы можете упростить задачу, разложив подкоренное число на квадратный множитель и обыкновенный множитель (число, из которого целый квадратный корень извлечь нельзя). Затем вы извлечете квадратный корень из квадратного множителя и будете извлекать корень из обыкновенного множителя.

    • Например, вычислите квадратный корень из числа 147. Число 147 нельзя разложить на два квадратных множителя, но его можно разложить на следующие множители: 49 и 3. Решите задачу следующим образом:
      • = √(49 х 3)
      • = √49 х √3
      • = 7√3
  • Если нужно, оцените значение корня. Теперь можно оценить значение корня (найти приблизительное значение), сравнив его со значениями корней квадратных чисел, находящихся ближе всего (с обеих сторон на числовой прямой) к подкоренному числу. Вы получите значение корня в виде десятичной дроби, которую необходимо умножить на число, стоящее за знаком корня.

    • Вернемся к нашему примеру. Подкоренное число 3. Ближайшими к нему квадратными числами будут числа 1 (√1 = 1) и 4 (√4 = 2). Таким образом, значение √3 расположено между 1 и 2. Та как значение √3, вероятно, ближе к 2, чем к 1, то наша оценка: √3 = 1,7. Умножаем это значение на число у знака корня: 7 х 1,7 = 11,9. Если вы сделаете расчеты на калькуляторе, то получите 12,13, что довольно близко к нашему ответу.
      • Этот метод также работает с большими числами. Например, рассмотрим √35. Подкоренное число 35. Ближайшими к нему квадратными числами будут числа 25 (√25 = 5) и 36 (√36 = 6). Таким образом, значение √35 расположено между 5 и 6. Так как значение √35 намного ближе к 6, чем к 5 (потому что 35 всего на 1 меньше 36), то можно заявить, что √35 немного меньше 6. Проверка на калькуляторе дает нам ответ 5,92 — мы были правы.
  • Еще один способ – разложите подкоренное число на простые множители . Простые множители – числа, которые делятся только на 1 и самих себя. Запишите простые множители в ряд и найдите пары одинаковых множителей. Такие множители можно вынести за знак корня.

    • Например, вычислите квадратный корень из 45. Раскладываем подкоренное число на простые множители: 45 = 9 х 5, а 9 = 3 х 3. Таким образом, √45 = √(3 х 3 х 5). 3 можно вынести за знак корня: √45 = 3√5. Теперь можно оценить √5.
    • Рассмотрим другой пример: √88.
      • = √(2 х 44)
      • = √ (2 х 4 х 11)
      • = √ (2 х 2 х 2 х 11). Вы получили три множителя 2; возьмите пару из них и вынесите за знак корня.
      • = 2√(2 х 11) = 2√2 х √11. Теперь можно оценить √2 и √11 и найти приблизительный ответ.

    Вычисление квадратного корня вручную

    При помощи деления в столбик
    1. Этот метод включает процесс, аналогичный делению в столбик, и дает точный ответ. Сначала проведите вертикальную линию, делящую лист на две половины, а затем справа и немного ниже верхнего края листа к вертикальной линии пририсуйте горизонтальную линию. Теперь разделите подкоренное число на пары чисел, начиная с дробной части после запятой. Так, число 79520789182,47897 записывается как «7 95 20 78 91 82, 47 89 70».

      • Для примера вычислим квадратный корень числа 780,14. Нарисуйте две линии (как показано на рисунке) и слева сверху напишите данное число в виде «7 80, 14». Это нормально, что первая слева цифра является непарной цифрой. Ответ (корень из данного числа) будете записывать справа сверху.
    2. Для первой слева пары чисел (или одного числа) найдите наибольшее целое число n, квадрат которого меньше или равен рассматриваемой паре чисел (или одного числа). Другими словами, найдите квадратное число, которое расположено ближе всего к первой слева паре чисел (или одному числу), но меньше ее, и извлеките квадратный корень из этого квадратного числа; вы получите число n. Напишите найденное n сверху справа, а квадрат n запишите снизу справа.

      • В нашем случае, первым слева числом будет число 7. Далее, 4
    3. Вычтите квадрат числа n, которое вы только что нашли, из первой слева пары чисел (или одного числа). Результат вычисления запишите под вычитаемым (квадратом числа n).

      • В нашем примере вычтите 4 из 7 и получите 3.
    4. Снесите вторую пару чисел и запишите ее около значения, полученного в предыдущем шаге. Затем удвойте число сверху справа и запишите полученный результат снизу справа с добавлением «_×_=».

      • В нашем примере второй парой чисел является «80». Запишите «80» после 3. Затем, удвоенное число сверху справа дает 4. Запишите «4_×_=» снизу справа.
    5. Заполните прочерки справа.

      • В нашем случае, если вместо прочерков поставить число 8, то 48 х 8 = 384, что больше 380. Поэтому 8 — слишком большое число, а вот 7 подойдет. Напишите 7 вместо прочерков и получите: 47 х 7 = 329. Запишите 7 сверху справа — это вторая цифра в искомом квадратном корне числа 780,14.
    6. Вычтите полученное число из текущего числа слева. Запишите результат из предыдущего шага под текущим числом слева, найдите разницу и запишите ее под вычитаемым.

      • В нашем примере, вычтите 329 из 380, что равно 51.
    7. Повторите шаг 4. Если сносимой парой чисел является дробная часть исходного числа, то поставьте разделитель (запятую) целой и дробной частей в искомом квадратном корне сверху справа. Слева снесите вниз следующую пару чисел. Удвойте число сверху справа и запишите полученный результат снизу справа с добавлением «_×_=».

      • В нашем примере следующей сносимой парой чисел будет дробная часть числа 780.14, поэтому поставьте разделитель целой и дробной частей в искомом квадратном корне сверху справа. Снесите 14 и запишите снизу слева. Удвоенным числом сверху справа (27) будет 54, поэтому напишите «54_×_=» снизу справа.
    8. Повторите шаги 5 и 6. Найдите такое наибольшее число на место прочерков справа (вместо прочерков нужно подставить одно и тоже число), чтобы результат умножения был меньше или равен текущему числу слева.

      • В нашем примере 549 х 9 = 4941, что меньше текущего числа слева (5114). Напишите 9 сверху справа и вычтите результат умножения из текущего числа слева: 5114 — 4941 = 173.
    9. Если для квадратного корня вам необходимо найти больше знаков после запятой, напишите пару нулей у текущего числа слева и повторяйте шаги 4, 5 и 6. Повторяйте шаги, до тех пор пока не получите нужную вам точность ответа (число знаков после запятой).

      Понимание процесса
      1. Для усвоения данного метода представьте число, квадратный корень которого необходимо найти, как площадь квадрата S. В этом случае вы будете искать длину стороны L такого квадрата. Вычисляем такое значение L, при котором L² = S.

        Задайте букву для каждой цифры в ответе. Обозначим через A первую цифру в значении L (искомый квадратный корень). B будет второй цифрой, C — третьей и так далее.

        Задайте букву для каждой пары первых цифр. Обозначим через S a первую пару цифр в значении S, через S b — вторую пару цифр и так далее.

        Уясните связь данного метода с делением в столбик. Как и в операции деления, где каждый раз нас интересует только одна следующая цифра делимого числа, при вычислении квадратного корня мы последовательно работаем с парой цифр (для получения одной следующей цифры в значении квадратного корня).

      2. Рассмотрим первую пару цифр Sa числа S (Sa = 7 в нашем примере) и найдем ее квадратный корень. В этом случае первой цифрой A искомого значения квадратного корня будет такая цифра, квадрат которой меньше или равен S a (то есть ищем такое A, при котором выполняется неравенство A² ≤ Sa

        • Допустим, что нужно разделить 88962 на 7; здесь первый шаг будет аналогичным: рассматриваем первую цифру делимого числа 88962 (8) и подбираем такое наибольшее число, которое при умножении на 7 дает значение меньшее или равное 8. То есть ищем такое число d, при котором верно неравенство: 7×d ≤ 8
      3. Мысленно представьте квадрат, площадь которого вам нужно вычислить. Вы ищите L, то есть длину стороны квадрата, площадь которого равна S. A, B, C — цифры в числе L. Записать можно иначе: 10А + B = L (для двузначного числа) или 100А + 10В + С = L (для трехзначного числа) и так далее.

        • Пусть (10A+B)² = L² = S = 100A² + 2×10A×B + B² . Запомните, что 10A+B — это такое число, у которого цифра B означает единицы, а цифра A — десятки. Например, если A=1 и B=2, то 10A+B равно числу 12.(10A+B)² — это площадь всего квадрата, 100A² — площадь большого внутреннего квадрата, — площадь малого внутреннего квадрата, 10A×B — площадь каждого из двух прямоугольников. Сложив площади описанных фигур, вы найдете площадь исходного квадрата.
  • Рекомендуем также

    Корень, Арифметический корень

    Давайте возьмем число 9. Девять делится на 3 и результат равен делителю 3 => 9/3 = 3, то есть 3.3 = 9 или 32 = 9.Давайте возьмем другое число, например 27, 27 = 3.3.3 = 33. Таким образом мы обнаружили, что 9 и 27 на самом деле являются числом 3 со степенью 2 и 3.

    В общем, арифметический корень (далее — корень) это функция, находящая делитель числа, который, будучи возведенным в степень корня, дает нам в результате снова это число. Иногда, этот делитель не является рациональным числом. В принципе корень — это обратная функция возведения в степень. Но даже может быть записан с помощью степени. Так, в нашем случае квадратный корень из 9 есть 3, √9 и кубический корень 27 есть 3 = 3√27

    Если a есть положительным действительным числом, тогда уравнение x2 = a имеет две решения: x = +√a or x = -√a.

    $\sqrt[2]{x} = \sqrt{x}$

    Если a есть действительным числом, тогда уравнение x3 = a имеет только одно решение => x = 3√a. \frac{1}{m\cdot n}$, или $\sqrt[n\cdot m]{a}$


    $\sqrt[2n]{x} \ge 0$   n — натуральное число (если x ≥ 0)

    Однообразие арифметического корня

    Если 0 ≤ x n√x n√y

    Функция квадратного корня
    Функция кубического корня

    Больше об арифметических корнях на страницах математического форума

    Извлечение квадратного корня из суммы или разности

    При работе с иррациональными числами наибольшие затруднения возникают в случае, когда необходимо извлечь корень из суммы или разности двух чисел: чаще одного рационального, а другого — иррационального. Рассмотрим на примере:


    Так как отсутствует формула, позволяющая разбить данный корень на разность двух корней, 

    мы воспользуемся следующими формулами: 

     и

    Таким образом, выделив под корнем три слагаемых и свернув их в формулу квадрата разности (в нашем случае), извлечем корень из квадрата, что упростит дальнейшие вычисления.

    Итак, начнем с иррационального числа, в нашем случае это -16√3. То, что число отрицательное, показывает нам, что сворачивать будем в квадрат разности. Число -16√3 содержит в себе удвоенное произведение двух чисел. Т.е. -16√3=-2ab. Это означает, что ab=8√3. Переберем все возможные варианты:

    1. 8√3=8*√3;
    2. 8√3=2*4√3;
    3. 8√3=4*2√3;
    4. 8√3=1*8√3.

    Для формулы необходимо к удвоенному произведению добавить сумму квадратов выражений a и b. Выполним это для каждого из вариантов и определим, в каком из случаев сумма будет составлять 28, чего требует условие.

    1. -2*8*√3+64+3=67-16√3;
    2. -2*2*4√3+4+48=52-16√3;
    3. -2*4*2√3+16+12=28-16√3;
    4. -2*1*8√3+1+192=193-16√3.

    Получается, что нам подходит третий случай, в котором выражение 28-16√3 раскладывается в формулу квадрата разности двух выражений: 4 и 2√3. Свернем по формуле и продолжим вычисления: 

    Теперь необходим раскрыть модуль, используя определения модуля. Для этого нужно определить знак подмодульного выражения. Т.к. 4>2√3 (чтобы узнать, какое число больше, можно просто возвести оба этих числа в квадрат. И т.к. квадрат 4 равен 16, а квадрат 2√3 равен 12 и 16>12, то и 4>2√3), то модуль раскрывается с тем же знаком, т.е. |4-2√3|=4-2√3.

    Для закрепления данной темы предлагаю вам самостоятельно извлечь корень из следующих выражений: 

    Формула квадратного корня

    — Что такое формула квадратного корня? Примеры

    Формула квадратного корня помогает представить любое число в форме его квадратного корня. Квадратный корень из любого числа — это то значение, которое при умножении само на себя дает исходное число. Он представлен с помощью символа «√». Каждое число имеет два квадратных корня, один с положительным значением, а другой с отрицательным. Давайте разберемся с формулой квадратного корня, используя решенные примеры в следующем разделе.

    Что такое формула квадратного корня?

    Квадратный корень из любого числа – это число, возведенное в степень 1/2.При вычислении квадратного корня из любого числа мы принимаем как отрицательные, так и положительные значения в качестве квадратного корня после вычисления. Формула квадратного корня для полного квадрата даст в результате целое число. Квадратный корень из отрицательного числа никогда не может быть действительным числом.

    Формула квадратного корня

    Формула квадратного корня числа x задается как  

    √х = х

    1/2

    Предположим, что x — это любое число, такое что x = y × y, формула для вычисления квадратного корня из x будет выглядеть так:

    √x = √(y × y) = y

    , где y — квадратный корень из любого числа x.Это также означает, что если значение y является целым числом, то x будет идеальным квадратом.

    Методы вычисления формул квадратного корня

    Несмотря на то, что существуют различные методы, которые можно удобно использовать для идеальных квадратов, метод деления в длину можно использовать для любого числа, независимо от того, является оно идеальным квадратом или нет.

    • Метод многократного вычитания квадратного корня
    • Извлечение квадратного корня методом простой факторизации
    • Квадратный корень по методу оценки
    • Извлечение квадратного корня методом деления в длину.

    Давайте посмотрим на несколько решенных примеров, чтобы лучше понять формулу квадратного корня.

     

    Разбивайте сложные концепции с помощью простых визуальных средств.

    Математика больше не будет сложным предметом, особенно когда вы понимаете концепции с помощью визуализаций.

    Забронируйте бесплатный пробный урок

    Примеры использования формулы квадратного корня

    Пример 1: Используя формулу квадратного корня, вычислите квадратный корень из 144.

    Решение:      

    Чтобы найти: квадратный корень из 144

    Из простой факторизации числа 144 получаем

    .

    144 = 2 × 2 × 2 × 2 × 3 × 3

    = (2 × 2 × 3) 2

    Используя формулу квадратного корня,

    √144 = ± [(2 × 2 × 3) 2 ] 1/ 2

    √144 = ±12

    Ответ: Квадратный корень из 144 = ±12

    Пример 2: Определите квадратный корень из 60.

    Решение:

    Чтобы найти: квадратный корень из 60

    Из простой факторизации числа 60 мы получаем

    .

    60 = 2 × 2 × 3 × 5

    = (2) 2 × 3 × 5

    Используя формулу квадратного корня,

    √60 = [(2) 2  × 15 ] 1/ 2

    √60 = 2√15

    Ответ: Квадратный корень из 60 = 2√15

    Пример 3: Вычислите длину стороны квадрата, площадь которого равна 400 квадратных единиц.

    Решение:

    Найти: Длина стороны квадрата.

    Дано, площадь квадрата = 400 кв. единиц

    Используя формулу квадратного корня или, точнее, формулу площади квадрата,

    Сторона = √(площадь) = √(сторона) 2

    = √400

    = 20 единиц

    Ответ: Длина стороны квадрата = 20 единиц

    Часто задаваемые вопросы о формуле квадратного корня

    Что такое формула квадратного корня в математике?

    В математике формула квадратного корня используется для представления любого числа в форме его квадратного корня, например, для любого числа x его квадратный корень будет √x = x 1/2

    Что такое формула квадратного корня для отрицательных чисел?

    Мы знаем, что у отрицательных чисел нет настоящих квадратных корней. Квадратные корни чисел, отличных от полного квадрата, считаются иррациональными числами. Главный квадратный корень любых отрицательных чисел, скажем -x, равен: √(-x)= i√x, где i – квадратный корень из -1.

    Каковы применения формулы квадратного корня?

    Существует множество применений формулы квадратного корня

    .
    • Используется в алгебре и геометрии. Он служит основой для формулы корней квадратного уравнения; квадратичные поля и кольца целых квадратичных чисел.
    • Часто используется во многих физических законах.
    • Для расчета площадей, объемов и других формул измерения.
    • Широко используется плотниками, архитекторами и инженерами.

    Как написать формулу квадратного корня словами?

    Квадратный корень из любого числа – это число, возведенное в степень 1/2.

    БиоМатематика: Квадратичные функции

    В этом разделе мы узнаем, как найти корень(и) квадратного уравнения.Корни также называют х — точками пересечения или нулями. Квадратичная функция графически изображается параболой с вершиной, расположенной в начале координат ниже оси x или выше оси x . Следовательно, квадратичная функция может иметь один, два или нулевые корни.

    Когда нас просят решить квадратное уравнение, на самом деле нас просят найти его корни. Мы уже видели, что завершение квадрата является полезным методом решения квадратных уравнений.Этот метод можно использовать для вывода квадратной формулы, которая используется для решения квадратных уравнений. На самом деле корни функции

    ф ( х ) = ах 2 + бх + с

    даются квадратичной формулой. Корни функции — это точки пересечения x . По определению координата y точек, лежащих на оси x , равна нулю. Следовательно, чтобы найти корни квадратичной функции, мы устанавливаем f ( x ) = 0 и решаем уравнение

    ах 2 + бх + с = 0.

    Мы можем сделать это, заполнив квадрат как,

    Решение для x и упрощение имеем,

    Таким образом, корни квадратичной функции задаются как,

    Эта формула называется квадратичной формулой , и ее вывод включен, чтобы вы могли видеть, откуда она взялась. Мы называем терм b 2 −4 ac дискриминантом .Дискриминант важен, потому что он говорит вам, сколько корней имеет квадратичная функция. В частности, если

    1. b 2 −4 ac < 0 Действительных корней нет.

    2. b 2 −4 ac = 0 Имеется один действительный корень.

    3. b 2 −4 ac > 0 Имеется два действительных корня.

    Мы рассмотрим каждый случай индивидуально.

    Случай 1: нет реальных корней

    Если дискриминант квадратичной функции меньше нуля, эта функция не имеет действительных корней, а парабола, которую она представляет, не пересекает ось x . Поскольку квадратичная формула требует извлечения квадратного корня из дискриминанта, отрицательный дискриминант создает проблему, потому что квадратный корень из отрицательного числа не определен относительно действительной прямой. Пример квадратичной функции без действительных корней:

    .

    ф ( х ) = х 2 — 3 х + 4.

    Обратите внимание, что дискриминант f ( x ) отрицателен,

    б 2 -4 а.с. = (-3) 2 — 4 · 1 · 4 = 9 — 16 = -7.

    Эта функция графически представлена ​​параболой, которая направлена ​​вверх, вершина которой лежит над осью x. Таким образом, график никогда не может пересекать ось x и не имеет корней, как показано ниже,

    Вариант 2: один реальный корень

    Если дискриминант квадратичной функции равен нулю, эта функция имеет ровно один действительный корень и пересекает ось x в одной точке. Чтобы увидеть это, мы устанавливаем b 2 −4 ac = 0 в квадратичной формуле, чтобы получить

    Обратите внимание, что это координата x вершины параболы. Таким образом, парабола имеет ровно один действительный корень, когда вершина параболы лежит прямо на оси x . Простейший пример квадратичной функции, имеющей только один действительный корень:

    .

    у = х 2 ,

    , где реальный корень равен x = 0.

    Другой пример квадратичной функции с одним действительным корнем дается выражением

    .

    f ( х ) = −4 х 2 + 12 х − 9,

    Обратите внимание, что дискриминант f ( x ) равен нулю,

    б 2 −4 ac = (12) 2 − 4 · −4 · −9 = 144 − 144 = 0,

    Эта функция графически представлена ​​параболой, которая раскрывается вниз и имеет вершину (3/2, 0), лежащую на оси x . Таким образом, график пересекает ось x ровно в одной точке (т.е. имеет один корень), как показано ниже,

     

     

    Случай 3: два действительных корня

    Если дискриминант квадратичной функции больше нуля, эта функция имеет два действительных корня ( x -пересечений). Извлечение квадратного корня из положительного действительного числа хорошо определено, а два корня равны

    .

    Пример квадратичной функции с двумя вещественными корнями:

    f ( х ) = 2 х 2 — 11 х + 5.

    Обратите внимание, что дискриминант f ( x ) больше нуля,

    b 2 − 4 ac = (−11) 2 − 4 · 2 · 5 = 121 − 40 = 81.

    Эта функция графически представлена ​​параболой, обращенной вверх, вершина которой лежит ниже оси x . Таким образом, график должен пересекать ось x в двух местах (т.е. иметь два корня), как показано ниже,

    *****

    В следующем разделе мы будем использовать квадратную формулу для решения квадратных уравнений.

    Решение квадратных уравнений

    Решение уравнений с квадратными корнями — Элементарная алгебра

    Цели обучения

    К концу этого раздела вы сможете:

    • Решение радикальных уравнений
    • Использование квадратных корней в приложениях

    Прежде чем начать, пройдите этот тест на готовность.

    1. Упрощение: ⓐ ⓑ .
      Если вы пропустили эту проблему, просмотрите (Рисунок) и (Рисунок).
    2. Решить: .
      Если вы пропустили эту проблему, просмотрите (рисунок).
    3. Решить: .
      Если вы пропустили эту проблему, просмотрите (рисунок).

    Решение радикальных уравнений

    В этом разделе мы будем решать уравнения, в которых переменная стоит под корнем квадратного корня. Уравнения такого типа называются радикальными уравнениями.

    Радикальное уравнение

    Уравнение, в котором переменная стоит под корнем квадратного корня, называется радикальным уравнением.

    Как обычно, при решении этих уравнений то, что мы делаем с одной частью уравнения, мы должны делать и с другой его частью. Поскольку возведение величины в квадрат и извлечение квадратного корня являются «противоположными» операциями, мы возведем обе стороны в квадрат, чтобы удалить знак радикала и найти переменную внутри.

    Но помните, что когда мы пишем, мы имеем в виду главный квадратный корень. Так всегда. Когда мы решаем радикальные уравнения, возводя обе части в квадрат, мы можем получить алгебраическое решение, которое будет отрицательным.Это алгебраическое решение не было бы решением исходного радикального уравнения; это постороннее решение . Мы видели посторонние решения и при решении рациональных уравнений.

    Для уравнения:

    ⓐ Есть решение? ⓑ Есть решение?

    Для уравнения:

    ⓐ Есть решение? ⓑ Есть решение?

    ⓐ нет ⓑ

    Для уравнения:

    ⓐ Есть решение? ⓑ Есть решение?

    ⓐ нет ⓑ

    Теперь посмотрим, как решить радикальное уравнение. Наша стратегия основана на соотношении между извлечением квадратного корня и возведением в квадрат.

    Как решать радикальные уравнения

    Решить: .

    Решить: .

    Решить: .

    Решите радикальное уравнение.

    1. Изолируйте радикал на одной стороне уравнения.
    2. Возведите в квадрат обе части уравнения.
    3. Решите новое уравнение.
    4. Проверьте ответ.

    Решить: .

    Решить: .

    Решить: .

    Решить: .

    Решить: .

    Решить: .

    Когда мы используем подкоренной знак, мы имеем в виду главный или положительный корень. Если уравнение имеет квадратный корень, равный отрицательному числу, то это уравнение не имеет решения.

    Решить: .

    Решение

    The figure then says, “To isolate the radical, subtract 1 from both sides.” Thus the equation becomes the square root of (9 k minus 2) plus 1 minus 1 equals 0 minus 1. The figure then says, “Simplify.” The equation is then the square root of (9 k minus 2 ) equals -1. The figure then says, “Since the square root is equal to a negative number, the equation has no solution.”» data-label=»»>
    Чтобы выделить радикал, вычтите 1 с обеих сторон.
    Упрощение.
    Поскольку квадратный корень равен отрицательному числу, уравнение не имеет решения.

    Решить: .

    Решить: .

    Если одна часть уравнения двучленная, мы используем формулу биномиальных квадратов, когда возводим ее в квадрат.

    Биномиальные квадраты

    Не забудьте средний термин!

    Решить: .

    Решить: .

    Решить: .

    Решить: .

    Решить: .

    Решить: .

    Если перед радикалом стоит коэффициент, его тоже нужно возвести в квадрат.

    Решить: .

    Решить: .

    Решить: .

    Решить: .

    Решение

    Решить: .

    Решить: .

    Иногда после возведения в квадрат обеих частей уравнения у нас все еще есть переменная внутри радикала. Когда это происходит, мы повторяем Шаг 1 и Шаг 2 нашей процедуры. Мы изолируем радикал и снова возводим в квадрат обе части уравнения.

    Решить: .

    Решение

    Решить: .

    Решить: .

    Решить: .

    Решение

    Решить: .

    Решить: .

    Использование квадратных корней в приложениях

    По мере прохождения курсов в колледже вы столкнетесь с формулами, содержащими квадратный корень, во многих дисциплинах. Мы уже использовали формулы для решения геометрических задач.

    Мы будем использовать нашу стратегию решения задач для приложений геометрии с небольшими изменениями, чтобы получить план решения приложений с формулами из любой дисциплины.

    Решение приложений с формулами.

    1. Прочтите задачу и убедитесь, что все слова и идеи понятны. При необходимости нарисуйте рисунок и подпишите его с помощью данной информации.
    2. Определите , что мы ищем.
    3. Назовите то, что мы ищем, выбрав переменную для ее представления.
    4. Переведите в уравнение, написав соответствующую формулу или модель для данной ситуации. Замените предоставленную информацию.
    5. Решите уравнение , используя хорошие методы алгебры.
    6. Проверьте ответ в задаче и убедитесь, что он имеет смысл.
    7. Ответьте на вопрос полным предложением.

    Мы использовали формулу, чтобы найти площадь прямоугольника с длиной L и шириной W . Квадрат – это прямоугольник, у которого длина и ширина равны. Если мы допустим, что s будет длиной стороны квадрата, площадь квадрата равна .

    Формула дает нам площадь квадрата, если мы знаем длину стороны.Что, если мы хотим найти длину стороны для данной площади? Затем нам нужно решить уравнение для s .

    Мы можем использовать формулу, чтобы найти длину стороны квадрата для данной площади.

    Площадь квадрата

    Мы покажем это в следующем примере.

    Кэти хочет посадить квадратную лужайку во дворе своего дома. У нее достаточно дерна, чтобы покрыть площадь в 370 квадратных футов. Используйте формулу, чтобы найти длину каждой стороны ее лужайки.Округлите ответ до ближайшей десятой доли фута.

    Серджио хочет сделать квадратную мозаику для украшения стола, который он собирает. У него достаточно плитки, чтобы покрыть площадь в 2704 квадратных сантиметра. Используйте формулу, чтобы найти длину каждой стороны его мозаики. Округлите ответ до ближайшей десятой доли фута.

    Другое применение квадратных корней связано с гравитацией.

    Падающие предметы

    На Земле, если объект падает с высоты фута, время в секундах, которое потребуется, чтобы достичь земли, определяется по формуле

    .

    Например, если объект падает с высоты 64 фута, мы можем найти время, необходимое для достижения земли, подставив в формулу.

    Извлеките квадратный корень из 64.
    Упростите дробь.

    Объекту, упавшему с высоты 64 фута, потребуется 2 секунды, чтобы достичь земли.

    Кристи уронила солнцезащитные очки с моста на высоте 400 футов над рекой. Используйте формулу, чтобы найти, сколько секунд потребовалось солнцезащитным очкам, чтобы достичь реки.

    Вертолет сбросил спасательный пакет с высоты 1296 футов. Используйте формулу, чтобы найти, сколько секунд потребовалось пакету, чтобы достичь земли.

    Мойщик окон уронил швабру с платформы на высоте 196 футов над тротуаром. Используйте формулу, чтобы найти, сколько секунд потребовалось швабре, чтобы добраться до тротуара.

    Полицейские, расследующие автомобильные аварии, измеряют длину следов заноса на тротуаре.Затем они используют квадратный корень, чтобы определить скорость в милях в час, которую двигала машина перед тем, как затормозить.

    Следы заноса и скорость автомобиля

    Если длина следов заноса составляет d футов, то скорость s автомобиля до включения тормозов можно найти по формуле

    После автомобильной аварии следы заноса одной машины составили 190 футов. Используйте формулу, чтобы найти скорость автомобиля до включения тормозов.Округлите ответ до десятых.

    Следователь ДТП измерил следы заноса автомобиля. Длина следов заноса составляла 76 футов. Используйте формулу, чтобы найти скорость автомобиля до включения тормозов. Округлите ответ до десятых.

    Следы заноса автомобиля, попавшего в аварию, имели длину 122 фута. Используйте формулу, чтобы найти скорость автомобиля до включения тормозов. Округлите ответ до десятых.

    Основные понятия

    • Для решения радикального уравнения:
      1. Изолируйте радикал на одной стороне уравнения.
      2. Возведите в квадрат обе части уравнения.
      3. Решите новое уравнение.
      4. Проверьте ответ. Некоторые полученные решения могут не работать в исходном уравнении.
    • Решение задач с помощью формул
      1. Прочтите задачу и убедитесь, что все слова и идеи понятны.При необходимости нарисуйте рисунок и подпишите его с помощью данной информации.
      2. Определите , что мы ищем.
      3. Назовите то, что мы ищем, выбрав переменную для ее представления.
      4. Переведите в уравнение, написав соответствующую формулу или модель для данной ситуации. Замените предоставленную информацию.
      5. Решите уравнение , используя хорошие методы алгебры.
      6. Проверьте ответ в задаче и убедитесь, что он имеет смысл.
      7. Ответьте на вопрос полным предложением.
    • Площадь квадрата
    • Падающие предметы
      • На Земле, если объект падает с высоты в фут, время в секундах, необходимое для достижения земли, определяется по формуле .
    • Следы заноса и скорость автомобиля
      • Если длина следов заноса составляет d футов, то скорость автомобиля до торможения s можно найти по формуле .
    Письменные упражнения

    Объясните, почему уравнение вида не имеет решения.

    1. ⓐ Решите уравнение.
    2. ⓑ Объясните, почему одно из найденных «решений» на самом деле не было решением уравнения.
    Самопроверка

    ⓐ После выполнения упражнений используйте этот контрольный список, чтобы оценить свое мастерство выполнения целей этого раздела.

    ⓑ Изучив этот контрольный список, что вы сделаете, чтобы стать уверенным в выполнении всех задач?

    Глоссарий

    радикальное уравнение
    Уравнение, в котором переменная стоит под корнем квадратного корня, называется радикальным уравнением

    Как вычислить квадратный корень вручную (с иллюстрациями)

    Краткое содержание статьиX

    Чтобы вычислить квадратный корень вручную, сначала оцените ответ, найдя 2 полных квадратных корня, между которыми находится число.Совершенный квадратный корень — это любой квадратный корень из целого числа. Например, если вы пытаетесь найти квадратный корень из 7, сначала вам нужно найти первый полный квадрат меньше 7, что равно 4, и первый правильный квадрат больше 7, что равно 9. Затем найдите квадратный корень из каждого полного квадрата. Квадратный корень из 4 равен 2, а квадратный корень из 9 равен 3. Следовательно, вы знаете, что квадратный корень из 7 находится где-то между 2 и 3. Теперь разделите полученное число на один из найденных вами идеальных квадратных корней.Например, вы бы разделили 7 на 2 или на 3. Если бы вы выбрали 3, ваш ответ был бы 2,33. Затем найдите среднее значение этого числа и идеальный квадратный корень. Чтобы найти среднее значение в этом примере, сложите 2,33 и 2, затем разделите на 2 и получите 2,16. Повторите процесс, используя среднее значение, которое вы получили. Сначала разделите число, из которого вы пытаетесь найти квадратный корень, на среднее значение. Затем найдите среднее значение этого числа и исходного среднего, сложив их вместе и разделив на 2. Например, сначала вы должны разделить 7, число, с которого вы начали, на 2.16, среднее значение, которое вы рассчитали, и получите 3,24. Затем вы должны добавить 3,24 к 2,16, старому среднему значению, и разделить на 2, чтобы найти новое среднее значение, равное 2,7. Теперь умножьте свой ответ сам на себя, чтобы увидеть, насколько он близок к квадратному корню из числа, с которого вы начали. В этом примере 2,7, умноженное само на себя, равно 7,29, что на 0,29 меньше 7. Чтобы приблизиться к 7, вы просто повторите процесс. Продолжайте делить число, с которого вы начали, на среднее значение этого числа и идеального квадрата, используя это число и старое среднее значение, чтобы найти новое среднее значение, и умножая новое среднее само на себя, пока оно не сравняется с вашим начальным числом.Если вы хотите узнать, как использовать алгоритм деления в длину для извлечения квадратного корня, продолжайте читать эту статью!

    Спасибо всем авторам за создание страницы, которую прочитали 2 287 317 раз.

    Извлечение квадратного корня без калькулятора

    You are here: Главная → Статьи → Алгоритм извлечения квадратного корня

    Большинство людей в современном мире считают, что, поскольку калькуляторы могут находить квадратные корни, детям не нужно учиться находить квадратные корни с помощью карандаша и бумаги.Однако изучение хотя бы метода «угадай и проверь» для нахождения квадратного корня на самом деле поможет учащимся ПОНЯТЬ и запомнить саму концепцию квадратного корня!

    Таким образом, даже если в вашем учебнике по математике может полностью отсутствовать тема нахождения квадратных корней без калькулятора, подумайте о том, чтобы позволить учащимся изучать и практиковать хотя бы метод «угадай и проверь». Поскольку на самом деле речь идет о КОНЦЕПЦИИ квадратного корня, я бы посчитал ее необходимой для изучения студентами.

    В зависимости от ситуации и учащихся, метод «угадай и проверь» можно использовать либо с помощью простого калькулятора, не имеющего кнопки извлечения квадратного корня, либо с помощью бумажных и карандашных вычислений.

     

    Нахождение квадратных корней методом угадывания и проверки

    Чтобы найти десятичное приближение, скажем, к √2, сначала сделайте начальное предположение, затем возведите его в квадрат и, в зависимости от того, насколько близко вы подошли, улучшите свое предположение.Поскольку этот метод включает в себя возведение в квадрат предположения (умножение числа на само себя), , он использует фактическое определение квадратного корня и поэтому может быть очень полезен при обучении понятию квадратного корня.


    Пример: чему равен квадратный корень из 20?

    Вы можете начать с того, что обратите внимание, что поскольку √16 = 4 и √25 = 5, то √20 должно быть между 4 и 5.

    Затем угадайте √20; скажем, например, что это 4,5. Возведите это в квадрат, посмотрите, будет ли результат больше или меньше 20, и улучшите свое предположение на основе этого.Повторяйте этот процесс, пока не получите желаемую точность (количество десятичных знаков). Это так просто и может стать хорошим экспериментом для студентов!


    Пример: Найти от √6 до 4 знаков после запятой

    Поскольку 2 2 = 4 и 3 2 = 9, мы знаем, что √6 находится между 2 и 3. Предположим (или оценим), что оно равно 2,5. Возводя это в квадрат, мы получаем 2,5 2 = 6,25. Это слишком много, поэтому мы немного уменьшаем нашу оценку. Давайте попробуем 2.4 дальше. Чтобы найти квадратный корень из 6 до четырех знаков после запятой, нам нужно повторять этот процесс, пока у нас не будет пять знаков после запятой, а затем мы округлим результат.

    Расчетный Площадь оценки Высокий / низкий
    2,4 5,76 Слишком низкое
    2,45 6,0025 Слишком высокая, но очень близко
    2,449 5,997601 Слишком мало
    2,4495 6,00005025 Слишком много, поэтому квадратный корень из 6 должен быть между 2.449 и 2.4495.
    2,4493 5,99
  • 9
  • Слишком низкое
    2,4494 5,99956036 Слишком низкое, так что квадратный корень из 6 должен быть между 2,4494 и 2,4495
    2,44945 5,9998053025 Слишком низкое, поэтому квадратный корень из 6 должен быть между 2,44945 и 2,4495.

    Этого количества итераций достаточно, поскольку теперь мы знаем, что √6 будет округлено до 2,4495 (а не до 2,4494).


    Нахождение квадратных корней с помощью алгоритма

    Существует также алгоритм извлечения квадратных корней, похожий на алгоритм деления в длину, и его преподавали в школах задолго до появления калькуляторов. См. пример ниже, чтобы узнать это. Хотя изучение этого алгоритма может не понадобиться в современном мире с калькуляторами, отработка некоторых примеров может быть использована в качестве упражнения в основных операциях для учащихся средней школы, а изучение лежащей в его основе логики может быть хорошим упражнением для мышления старшеклассников.


    Пример: Найдите √645 с точностью до одного десятичного знака.

    Сначала сгруппируйте числа под корнем попарно справа налево, оставив либо одна, либо две цифры слева (6 в данном случае). Для каждой пары чисел вы получите одну цифру квадратного корня.

    Для начала найдите число квадрат которого меньше или равен первой паре или первому числу, и напишите над линией квадратного корня (2):

    Затем продолжайте в том же духе:

    7 0,45
    2
    6 .45
     — 4
    (4 _) 2 85 91 049
    2
    6 √
    — 4
    (45) 2 45
    Возведите в квадрат 2, давая 4, напишите, что под 6 и вычесть.Сбить следующую пару цифр. Затем удвойте число над квадратным корнем линия символов (выделена) и запишите его в скобках с пустой строкой рядом с ним, как показано. Далее подумайте, какое однозначное число может что-то перейти на пустую строку так, чтобы сорок- что-то умножить на что-то быть меньше или равно 245.
    45 х 5 = 225
    46 х 6 = 276, поэтому 5 работает.
    2

    5

    6 .45 . 00
    — 4
    (45) 2 45
    — 2 25
    20 00
    2

    5

    6 .45 . 00
    — 4
    (45) 2 45
    — 2 25
    (50_) 20 00
    2

    5

    . 3
    6 .45 . 00
    — 4
    (45) 2 45
    — 2 25
    (503) 20 00
    Напишите 5 в начале строки.Вычислите 5 х 45, напишите, что меньше 245, вычтите, вывести следующую пару цифр (в данном случае десятичные цифры 00). Затем удвойте число над строкой (25) и напишите удвоенное число (50) в скобках с пустой строкой рядом с ним как указано: Подумай что однозначный номер что-то может пойти на пустом месте очередь чтоб пятьсот- что-то раз что-то было бы меньше или равно 2000.503 х 3 = 1509
    504 х 4 = 2016, поэтому 3 работает.
    2

    5

    . 3
    6 .45 . 00 .00
    — 4
    (45) 2 45
    — 2 25
    (503) 20 00
    —  15 09

    4

    91 00
    2

    5

    . 3
    6 .45 . 00 .00
    — 4
    (45) 2 45
    — 2 25
    (503) 20 00
    —  15 09
    (506_) 

    4

    91 00
    2

    5

    . 3 9
    6 .45 . 00 .00
    — 4
    (45) 2 45
    — 2 25
    (503) 20 00
    —  15 09
    (506_)  4 91 00
    Вычислите 3 х 503, напишите, что ниже 2000, вычесть, опустить следующие цифры. Затем удвойте «число» 253, которое находится над линией (игнорируя десятичную точку), и напишите удвоенное число 506 в скобках с пустой строкой рядом с ним как указано: 5068 х 8 = 40544
    5069 х 9 = 45621, что меньше 49100, поэтому 9 работает.

    Таким образом, до одного знака после запятой, 645 = 25.4

    Комментарии посетителей

    Я смутно припоминаю, что изучал алгоритм квадратного корня в К-12, но, честно говоря, я не вижу в этом алгоритме никакой ценности, кроме любопытства. И я не из «реформаторской» толпы. Я полностью уверен, что учащимся не дадут калькулятор для использования до продвинутой алгебры или предварительного исчисления, а затем только научный калькулятор (не графический). Вы действительно верите, что ученик уровня К-7 поймет, как/почему работает этот алгоритм?

    Я был рад видеть, что вы рекомендовали метод «оценить и проверить».Это то, что я также рекомендовал своей дочери, которая сейчас изучает квадратный корень по школьной программе. Метод «оцени и проверь» — хорошее упражнение для оценки, умножения, а также запоминания правильных квадратов.

    Другим методом, более подходящим для студентов, изучающих алгебру, было бы упрощение радикала с помощью общепринятого метода. Затем найдите оставшийся квадратный корень с помощью метода оценки. Например, чтобы найти SQRT (1400), упростите его до SQRT (100) * SQRT (14), что равно 10 * SQRT (14).Затем найдите SQRT(14) методом оценки. Для квадратных корней из полных квадратов даже не потребуется оценка.

    Можно даже превратить задачу нахождения квадратных корней в упражнение по компьютерному программированию, попросив учащихся написать программу на JavaScript или другом языке, чтобы использовать систематический числовой метод оценки этого квадратного корня с помощью метода проверки и предположения. Или, на уровне исчисления, студент может написать программу, которая использует многочлен Тейлора для вычисления квадратного корня.

    Майкл Саковски
    Преподаватель математики


    Привет,

    Заметил несколько комментариев, связанных с использованием алгоритма поиска квадратный корень числа. В некоторых комментариях говорилось, что поиск результата с помощью бумаги и ручки по сравнению с калькулятором является архаичным. Тот Может быть, так. Однако, когда я был на первом курсе средней школы (начало 70-х) Герр Куиннелл упомянул — когда класс выходил — некоторые вещей, которые можно делать с математикой, включая нахождение квадратных корней.Итак, я спросил его, как это было сделано. Он показал мне метод алгоритма на борту.

    Я не могу говорить о ценности общего знания того, как это используется в других профессии. В электронике нахождение квадратного корня является неотъемлемой частью часть дизайна. У нас есть детали, называемые резисторами. Они помогают ограничить ток в цепях. Эти детали имеют номинальную мощность. Значение резистора измеряется в «Омах». В математическом смысле это можно найти, разделив вольты по амперам.10 вольт разделить на 0,001 ампер — это сопротивление 10000 Ом. В качестве примера квадратного корня, если я знаю, что резистор на 10 000 Ом имеет мощность 0,25 Вт Я могу рассчитать максимальное наихудшее напряжение, которое может появиться на нем, до того, как может произойти повреждение. Это можно найти, взяв сопротивление значение — умножение номинальная мощность — и нахождение квадратного корня. Квадратный корень из 2500 равен 50. Эта часть могла выдержать 50 вольт.

    Моя точка зрения — я мог бы рассчитать результат, используя «искусственные средства».Так как кто-то нашел время, чтобы показать мне, как извлекать квадратный корень на доске, Мне не нужно было искать калькулятор. К тому времени я бы нашел калькулятор Я уже понял ответ. Найдите время, чтобы показать студентам то, как делаются такие вещи, как квадратный корень, имеет значение. На самом деле они могут не поставить это использовать позже в жизни — но некоторые просто могут.

    Гарт Прайс, CET


    Я как раз писал еще один комментарий, и каким-то образом компьютер отправил его до того, как я закончил.Должно быть, я нажал не ту клавишу. Итак, позвольте мне закончить, сказав, что дети плохо знакомы с миром и изучают его. Я считаю, что вычисление квадратных корней вручную было бы для них увлекательным и отличным способом узнать о других темах математики. Да, и, кстати, у меня вообще не было уроков квадратных корней до старшей школы, и тогда мы не научились их вычислять. Нас учили разлагать числа под радикалом и извлекать правильные квадраты, оставляя совершенные квадраты под радикалом.ПОТОМУ ЧТО ДАЖЕ УЧИТЕЛЬ НЕ ЗНАЛ, КАК ЭТО СДЕЛАТЬ ПРАВИЛЬНО.

    Роберт Монро


    это один из самых лучших сайтов, которые я посетил для правильного процесса решения проблемы. Вы можете называть меня архаиком, но когда я ходил в школу, они учили делению на две части, чтобы находить квадратный корень из числа.
    ГЛАВНОЕ, ЭТО УЧИТ ДУМАТЬ. Использование калькулятора — это форма чистой лени. Я чувствую, что наши дети думают, что получение основ в школе (РАНЬШЕ) является архаичным.Вот почему, когда вы идете в магазин и получаете счет на 16,75, и вы даете кассиру двадцатидолларовую купюру, однодолларовую купюру и 75 центов, они понятия не имеют, какая сдача должна быть, пока кассовый аппарат не скажет им, сколько чтобы дать тебе. Это приводит к ЛЕНИ ДУМАТЬ ИЛИ ВООБЩЕ ОТСУТСТВИЮ ДУМАТЬ.
    Спасибо за уделенное время.

    Раш Керлин


    Я искал в Интернете давно забытую процедуру поиска квадратных корней вручную и наткнулся на вашу веб-страницу. и хотел сказать, что многие (или все) критики стандартного алгоритма называют его «архаичным», «тупиковым» методом и т. д.в пользу вавилонского метода не может быть оправдано. Дело в том, что использование бумаги и карандаша для деления в длину или извлечения квадратных корней является архаичным и тупиковым процессом в 21 веке, независимо от того, какую процедуру мы используем, поскольку мы больше не делаем этого для каких-либо практических целей. расчеты. Итак, вопрос в том, чему мы должны учить студентов, чтобы познакомить их с фундаментальными методами? Вавилонский метод — это численный метод, в отличие от другого метода, и имеет смысл обучать сначала стандартной процедуре, которая работает с любыми числами, а затем другим приблизительным численным методам, а не использовать численные методы типа «предиктор-корректор», говоря, что они применимы где-то еще.Если мы используем методы типа «предиктор-корректор», необходимо также провести анализ ошибок, который не требуется при стандартном методе, поскольку при стандартной процедуре правильные цифры добавляются одна за другой на каждом шаге (в отличие от вавилонского метода, где содержание цифр может изменяться при каждом усреднении).

    С наилучшими пожеланиями,
    Карл И. Джейкоб
    Профессор факультета полимерной, текстильной и волокнистой инженерии
    Профессор Школы машиностроения им. Г. В. Вудраффа
    Технологический институт Джорджии


    Вы предоставили ответ на адрес «Нахождение квадратных корней с использованием алгоритма».Я заметил, что предоставленный ответ был оспорен несколькими людьми по нескольким причинам. Я хотел бы отметить, что предоставленное решение является самым старым методом решения квадратных корней в западном мире. Я был описан Леонардо Пикано, также известным как Фибоначчи, в его книге Liber Abaci, глава 14. Первое издание было «написано» в 1202 году, а второе издание было «написано» в 1228 году. Я говорю «написано», потому что оно было буквально написаны от руки, как и все копии. Работа Иоганна Гутенберга над печатным станком началась только в 1436 году.
    Леонардо научился этому методу во время своих арабских путешествий по Средиземному морю, а арабы узнали его от индуистской нации в современной Индии. Метод в показанном вами примере включает в себя некоторую современную интерпретацию, которая облегчает чтение. Леонардо также показал геометрическую взаимосвязь, связанную с тем, что мы сегодня понимаем как «аккорды». Это очень простое, некалькуляторное решение вопроса.

    Дэвид Т. Кэрротт, доктор философии


    Я прочитал ваше предложение по вычислению квадратного корня без калькулятора.Я преподаю математику для учителей начальных классов и курсы развивающей математики (алгебра) для взрослых. Я чувствую, что основное внимание следует уделять пониманию числа, а не упражнению в следовании заученным алгоритмам. Я предлагаю вам предложить ученику определить пару правильных квадратов, между которыми находится число. Например, при нахождении sqrt числа 645 оно попадает между sqrt 625, что равно 25, и sqrt 676, что равно 26. Таким образом, sqrt 645 должен быть между 25 и 26. Где оно находится между? Между 676 и 625 находится 50 чисел.645 — это 20 чисел после 625, поэтому 20/50 = 0,4. Таким образом, sqrt числа 645 очень близок к 25,4
    . Этот метод предоставляет учащимся процесс, который улучшает их понимание чисел, не ожидая от них запоминания алгоритма, и дает ответ с точностью до десятых.

    Андреа С. Леви, Эд.Д.


    В настоящее время я учусь в MCC. Я прохожу курс для учителей начальной математики. Мы должны составить план урока, чтобы научить детей начальных классов пользоваться теоремой Пифагора.Мне нужно научиться разбирать теорему Пифагора для младшего ребенка. Я застрял в части квадратного корня.

    Прочитайте мой ответ на этот вопрос.


    Метод, который вы показываете в статье, архаичен. Есть НАМНОГО более эффективный алгоритм. (Этот алгоритм фактически используется за кулисами внутри калькулятора, когда вы нажимаете кнопку извлечения квадратного корня.)

    1. Вычислите квадратный корень хотя бы из 1 цифры.
    2. Разделите эту оценку на число, квадратный корень которого вы хотите найти.
    3. Найдите среднее значение частного и делителя. Результат становится новой оценкой.

    Прелесть этого метода в том, что точность оценки растет чрезвычайно быстро. Каждый цикл по существу удваивает количество правильных цифр. Из начальной точки с 1 цифрой вы можете получить 4-значный результат за два цикла. Если вы уже знаете квадратный корень до нескольких цифр, например, sqrt(2)=1,414, один цикл деления и среднего даст вам удвоение цифр (в данном случае восемь).

    Помимо того, что этот метод дает возможность находить квадратные корни вручную, этот метод можно использовать, если все, что у вас есть, это дешевый калькулятор с 4 функциями. Если учащиеся смогут извлекать квадратные корни вручную, они не сочтут квадратные корни такими уж загадочными. Также этот метод является хорошим первым примером итеративного решения задачи.

    Дэвид Чендлер

    Этот другой способ называется Вавилонским методом угадывания и деления, и он действительно быстрее. Это также то же самое, что и при применении метода Ньютона.Посмотрите, например, нахождение квадратного корень из 20, используя 10, как первоначальный Угадка:

    Угадайте Divide Найти в среднем
    10 20/10 = 2 Среднее 10 И 2, чтобы дать новое предположение о 6
    6 20/6 = 3.333 20/6 = 3.333 Среднее 3,333 и 6 дает 4,6666
    4,666 20 / 4.666 = 4.1414 Среднее 4.= 4,4048 666,4.1414
    4,4048 20 / 4,4048 = 4,5454 средний = 4,4700
    4,4700 20 / 4,4700 = 4,4742 средний = 4,4721
    4,4721 20 / 4,4721 = 4.47217 Среднее значение = 4.47214
    Это уже до 4 десятичных знаков
    4,47214 20 / 4.47214 = 4,472132 Средние = 4.472135
    4.472135 20/4,472135=4,472137 среднее = 4,472136

    Плакат утверждает, что метод статьи «архаичен» и что «вавилонский метод» более эффективен. На первый взгляд может показаться, что это так, потому что пример автора находит квадратный корень из двузначного целого числа 20 вместо примера статьи 645.

    Тем не менее, я фактически разработал пример статьи (квадратный корень из 645), используя оба метода, и обнаружил, что вавилонский метод требует 9 «циклов деления и среднего», чтобы получить ответ.Кроме того, вавилонский метод требует от ученика деления на 5 цифр, что немаловажно для ученика начальной или средней школы. Метод статьи, с другой стороны, требует, чтобы студент выполнил только одну 4-шаговую задачу на длинное деление, решив не более полудюжины или около того задач на умножение 4 x 1 цифры.

    Таким образом, разумно заключить, что вавилонский метод больше подходит для решения с помощью калькулятора или компьютера, в то время как метод, описанный в статье, больше подходит для решения с помощью карандаша и бумаги.

    Поскольку предметом статьи было то, как научить ученика начальной или средней школы легко находить квадратные корни с помощью карандаша и бумаги, «архаичный» метод статьи кажется наиболее подходящим.

    Алекс


    В ответ на сообщение Алекса, как вам потребовалось 9 циклов, чтобы получить 25,4 с использованием вавилонского метода на 645? Это займет 1,5 шага, если вы используете свое предположение как 25
    1) 645/25 = 25,8
    (25 + 25,8)/2 = 25,4

    2) 645/25.4 ≈ 25,39

    Вавилонский метод очень эффективен, если уже известно много полных квадратов для аппроксимации исходного значения. Я обнаружил, что студенты не могут понять причины, лежащие в основе алгоритма в этом посте, в то время как метод деления и среднего кажется более интуитивным, если они раньше работали со средними значениями.

    Дэниел


    Я сомневаюсь в преподавании метода деления в длину для извлечения квадратных корней. Вавилонский метод легче запомнить и понять, и он дает столько же практики в основах арифметики.Что еще более важно, он имеет четкие связи с такими темами, как метод Ньютона и рекурсивные последовательности, которые будут встречаться в исчислении и за его пределами. Метод длинного деления несколько быстрее для ручного расчета, но он не ведет к другим важным темам — это тупик.

    Дэвид


    Меня обучали работе со старыми компьютерными схемами и бинарными аппаратными алгоритмами. Метод, используемый для вычисления корня из 645, является методом, используемым в высокопроизводительных двоичных вычислениях, поскольку он требует только сдвига, вычитания и сравнения, которые представляют собой инструкции одного цикла/этапа или направляются на сопроцессор.Преобразуйте число в двоичное, разбейте его на 2 битовые группы и используйте приведенную выше процедуру. Умножение и деление требуют от десятков до сотен циклов/стадий и убивают преформирование и конвейеры. Извлечение квадратного корня выполняется быстрее, чем деление, поскольку деление работает через 1 бит за цикл/этап, а квадратный корень шагает через 2 бита за цикл.

    Брэд



    чему равен корень квадратный из -1?

    Тамара Ярдли

    -1 не может иметь квадратного корня (по крайней мере, не вещественного), потому что любые два числа с одинаковым «знаком» (+/- положительный или отрицательный) при умножении дадут положительное число.Попробуйте: +2 × +2 = 4    и     -2 × -2 = 4.

    Поскольку квадратный корень числа должен равняться этому числу при умножении на себя. Когда вы умножаете это число само на себя и представляете его как полное уравнение ( n * n = x ), два множителя (n и n) либо оба положительны, либо оба отрицательны, поскольку они являются одним и тем же числом. Следовательно, их произведение будет положительным. Никакое действительное число, умноженное само на себя, не будет равняться отрицательному числу, поэтому -1 не может иметь действительный квадратный корень.

    Блейк

    Квадратный корень из -1 не является действительным числом.Она обозначается i и называется мнимой единицей. Из i и его кратных мы получаем чисто мнимые числа, такие как 2i, 5,6i, -12i и так далее. Это приводит к совершенно новой системе счисления комплексных чисел, в которой числа имеют действительную часть и мнимую часть (например, 5 + 3i или -20 — 40i). И с этой системой счисления делается много увлекательной математики!


    Я пытался найти в сети старый способ извлечения квадратных корней путем деления на длинные. ДА, я нашел это.Прочитайте ответы и не согласитесь со многими плакатами.

    Найти квадрат 645 легко, если вы знаете 252 и 262, но я никогда не запоминал квадраты чисел от 1 до 30 или около того, я запоминал только до 12X12 (старая имперская система)

    Угадай, что квадрат 645 равен примерно 25, это здорово, но если ты угадаешь, что это 2, то у тебя впереди более серьезная проблема.

    Я вижу, что «другие» плакаты находят более простые и быстрые способы… вот в чем сегодня беда. Давайте искать легкий путь без понимания.С вашим методом это может сделать любой, у кого есть деление в длину и простые навыки умножения. Самое простое решение — купить калькулятор и отказаться от всех умственных способностей. ржу не могу

    квадратный корень из 645 мммм 20
    645/20 = 32,25 в среднем 52,25 = 26,25
    645/26,25 = 24,57 среднее из 50,82 = 25,41

    Метод усреднения, кажется, работает, но он не учит многому делению… что-то вроде того, что выше/ниже в «Цена правильная».

    Я предполагаю, что квадрат 645 равен 25.41….вау работает с первого раза, чему я научился, ничего.

    Используя метод усреднения, определите квадратный корень из 9331671….моя первая оценка — 10, получайте удовольствие!
    9331671/10 = 933167,1 + 10 = 9331681,1/2 = 466588,55
    9331671/466588,55 = 19,999785 + 466588,55 = 466607,57/2 = 233303,285
    9331671/233303,285 = 39,99802 + 233303,285 = 233343,27/2 = 116671,235





    Ах да, это дети в 3 или 4 классе, занимающиеся длинной математикой с 8-значными числами…так много для усреднения. И какова степень значимости, поскольку мы работаем с одним десятичным знаком или с 3 …. не хотим «усреднять» слишком рано, иначе мы можем потерять значащие цифры. Если мы работаем с миллиардами, слишком быстрое удаление цифр может иметь ОГРОМНОЕ значение.

    Адриан


    Я дилетант, который пришел на сайт через поиск Google по теме «как вычислить квадратный корень». Я прочитал презентацию, потом посмотрел ответы. Я должен сказать, что был встревожен комментарием, предложенным Андреа С.Леви, Эд. Д., где она предположила, что запоминание алгоритма менее желательно, чем понимание числа.

    В настоящее время я работаю техническим писателем в фирме, которая пишет программное обеспечение для кредитных союзов. Понимание всех алгоритмов, используемых в финансовом мире, крайне важно для нас, чтобы делать то, что мы делаем. На самом деле, один из расчетов, который мы используем для определения амортизации потребительского кредита с комиссией за определенный период времени, поразительно похож на ваше представление квадратного корня. Расчет должен быть написан инженером-программистом для машины, поэтому в конечном итоге он находится в уме человека.Если инженер не знает алгоритма, последствия понесут тысячи потребителей. Я полагаю, что запоминание — это просто еще один инструмент в коробке. Используйте его, когда это уместно.

    С уважением,
    Майкл Келли
    Ньюбери-Парк, ок.


    Последний комментатор на странице (Адриан) сказала, что она никогда не выучила квадраты от 1 до 30. Это напоминает трюк, который я недавно выучил для нахождения квадратов, близких к 50. Начните с квадрата 50, 2500, прибавьте 100 раз расстояние между 50 и числом, а затем добавить квадрат расстояния 50 и число.Например, 43 2 = 2500 — 700 + 49 = 1849. Это происходит из простого тождества FOIL (50 + x) 2 = 2500 — 100x + x 2 . В этом тождестве x — это расстояние между 50 и числом. Если число равно 43 (как в моем примере), x равно -7. Если число равно 54, x равно 4. Итак, если вы запомните свои квадраты от 1 до 25, вы получите квадраты от 26 до 75 «бесплатно».

    Если идея запоминания квадратов от 1 до 25 кажется пугающей, то это не так. Несколько недель назад, прежде чем я узнал об этом трюке, я знал только до 13 навскидку, с несколькими другими, разбросанными здесь и там.Я составил таблицу в Excel, перечислив числа от 1 до 25 рядом с их квадратами, распечатал ее и повесил на стену в своей кабинке. Квадраты, которые я не запомнил в этих первых 25, я теперь могу получить за несколько секунд (например, для квадрата 23 я все еще считаю от 20 в квадрате: 400, 441, 484, *529*). Даже не совсем зная их все, я могу найти квадраты от 1 до 75 менее чем за 10 секунд (мыслительный процесс для нахождения 73 в квадрате навскидку: «73 — это 23 больше, чем 50. Что снова 23 в квадрате? 400, 441, 484, 529! 2500 + 2300 + 529 = 5329.Сделанный!»)

    Дэвид Леви


    См. также

    Еще один пример использования алгоритма извлечения квадратного корня

    Объяснение того, почему этот алгоритм квадратного корня работает.{2}=a\cdot a=\left (-a \right)\cdot \left (-a \right)$$

    Квадратный корень записывается с подкоренным символом √, а число или выражение внутри подкоренного символа, обозначенное ниже a, называется подкоренным символом.

    $$\sqrt{a}$$

    Чтобы указать, что нам нужен как положительный, так и отрицательный квадратный корень из подкоренного числа, мы помещаем символ ± (читается как плюс минус) перед корнем.

    $$\pm \sqrt{9}=\pm 3$$

    Ноль имеет один квадратный корень, равный 0.

    $$\sqrt{0}=0$$

    Отрицательные числа не имеют действительных квадратных корней, так как квадрат либо положителен, либо равен 0.

    Если квадратный корень из целого числа является другим целым числом, то квадрат называется полным квадратом.Например, 25 — это полный квадрат, начиная с

    .

    $$\pm \sqrt{25}= \pm 5$$

    Если подкоренное число не является полным квадратом, т.е. квадратный корень не является целым числом, вам необходимо аппроксимировать квадратный корень

    $$\pm \sqrt{3}= \pm 1.73205…\приблизительно \pm 1.7$$

    Квадратные корни чисел, не являющихся полными квадратами, являются членами иррациональных чисел. Это означает, что их нельзя записать как частное двух целых чисел. Десятичная форма иррационального числа не будет ни оканчиваться, ни повторяться.Иррациональные числа вместе с рациональными числами составляют действительные числа.


    Видеоурок

    Приблизительно квадратный корень из 250

    Калькулятор квадратного корня

    Использование калькулятора

    Используйте этот калькулятор, чтобы найти главный квадратный корень и корни действительных чисел. Входными данными для подкоренного числа x могут быть положительные или отрицательные действительные числа.Ответ также скажет вам, вошли ли вы в идеальный квадрат.

    Ответ покажет вам сложные или мнимые решения для квадратных корней из отрицательных действительных чисел. См. также Simplify Radical Expressions Calculator для упрощения радикалов вместо поиска дробных (десятичных) ответов.

    Квадратные корни, нечетные и четные:

    Есть 2 возможных корня для любого положительного действительного числа.Положительный корень и отрицательный корень. Дан номер x , квадратный корень из x — это число a такое, что а 2 = х . Квадратные корни — это специализированная форма нашего общего калькулятор корней.

    «Обратите внимание, что любое положительное действительное число имеет два квадратных корня, один положительный и один отрицательный. Например, квадратные корни из 9 равны -3 и +3, поскольку (-3) 2 = (+3) 2 = 9.Любое неотрицательное действительное число x имеет уникальный неотрицательный квадратный корень r; это называется главным квадратным корнем ………. Например, главный квадратный корень из 9 равен sqrt(9) = +3, а другой квадратный корень из 9 равен -sqrt(9) = — 3. В обычном использовании, если не указано иное, «квадратный корень» обычно означает главный квадратный корень.»[1].

    Калькулятор идеального квадрата

    Этот калькулятор также сообщит вам, является ли введенное вами число правильным квадратом или нет.Полный квадрат — это число x , где квадратный корень из x представляет собой число a , такое что a 2 = x и a является целым числом. Например, 4, 9 и 16 являются полными квадратами, поскольку их квадратные корни, 2, 3 и 4, соответственно, являются целыми числами.

    Пример квадратных корней:

    • Второй корень из 81, или 81 радикал 2, или квадратный корень из 81 записывается как $$ \sqrt[2]{81} = \sqrt[]{81} = \pm 9 $$.
    • Второй корень из 25, или 25 радикал 2, или квадратный корень из 25 записывается как $$ \sqrt[2]{25} = \sqrt[]{25} = \pm 5 $$.
    • Второй корень из 100, или 100 радикал 2, или квадратный корень из 100 записывается как $$ \sqrt[2]{100} = \sqrt[]{100} = \pm 10 $$.
    • Второй корень из 10, или 10 радикал 2, или квадратный корень из 10 записывается как $$ \sqrt[2]{10} = \sqrt[]{10} = \pm 3,162278 $$.

    Для расчета степени дроби используйте наш калькулятор Дробные показатели.

    Ссылки

    [1] Вайсштейн, Эрик В. «Квадратный корень». От MathWorld — Веб-ресурс Wolfram. Квадратный корень

    Дополнительное чтение по квадратным корням:

    В математике весело: квадратный корень

    .